Download as docx, pdf, or txt
Download as docx, pdf, or txt
You are on page 1of 142

UNIVERSITY OF MINDANAO

College of Engineering Education


Computer Engineering Program

Physically Distanced but Academically Engaged

Self-Instructional Manual (SIM) for Self-Directed Learning (SDL)

CEE 101- Differential Calculus for Engineering


Engr. Kimberly M. Nepa
Authors:
ENGR. JOHN A. BACUS
ENGR. MARIANNE WATA

THIS SIM/SDL MANUAL IS A DRAFT VERSION ONLY; NOT FOR


REPRODUCTION AND DISTRIBUTION OUTSIDE OF ITS INTENDED
USE. THIS IS INTENDED ONLY FOR THE USE OF THE STUDENTS
WHO ARE OFFICIALLY ENROLLED IN THE COURSE/SUBJECT.
EXPECT REVISIONS OF THE MANUAL.
College of Engineering Education
2nd Floor, BE Building
Matina Campus, Davao City
Telefax: (082)296-1084
Phone No.: (082)300-5456/300-0647 Local 131

TABLE OF CONTENTS

PAGE
Cover Page ………………………………………………………………………………………………. 1
Table of Contents………………………………………………………………………………………. 2
Course Outline…………………………………………………………………………………………... 7
Course Outline Policy………………………………………………………………………………… 7
Course Information…………………………………………………………………………………… 10

Topic/ Activity
Unit Learning Outcomes- Unit 1 (Weeks 1-3)……………………………………………… 11
Big Picture in Focus: ULO-1a…………………………………………………………………..….. 11
Metalanguage…………………………………………………………………………………... 11
Essential Knowledge………………………………………………………………………… 12
Self-Help………………………………………………………………………………………….. 17
In a Nutshell…………………………………………………………………………………….. 20
Q & A List ……………………………………………………………………………………...… 20
Big Picture in Focus: ULO-1b…………………………………………………………………..….. 21
Metalanguage………………………………………………………………………………….. 21
Essential Knowledge………………………………………………………………………… 22
Self-Help………………………………………………………………………………………….. 24
In a Nutshell…………………………………………………………………………………….. 25
Q & A List ………………………………………………………………………………………... 26
Big Picture in Focus: ULO-1c…………………………………………………………………..….. 26
Metalanguage…………………………………………………………………………………... 26
Essential Knowledge………………………………………………………………………… 27
Self-Help………………………………………………………………………………………….. 32
In a Nutshell…………………………………………………………………………………….. 32
Q & A List ……………………………………………………………………………………...… 33

2
College of Engineering Education
2nd Floor, BE Building
Matina Campus, Davao City
Telefax: (082)296-1084
Phone No.: (082)300-5456/300-0647 Local 131

PAGE
Unit Learning Outcomes- Unit 2 (Weeks 4-5) …………………………………………………. 34
Big Picture in Focus: ULO-2a…………………………………………………………………..….. 34
Metalanguage…………………………………………………………………………………... 34
Essential Knowledge………………………………………………………………………… 34
Self-Help………………………………………………………………………………………….. 37
In a Nutshell…………………………………………………………………………………….. 39
Q & A List ………………………………………………………………………………………... 39
Big Picture in Focus: ULO-2b…………………………………………………………………..….. 40
Metalanguage…………………………………………………………………………………... 40
Essential Knowledge………………………………………………………………………… 40
Self-Help………………………………………………………………………………………….. 44
In a Nutshell…………………………………………………………………………………….. 46
Q & A List ………………………………………………………………………………………... 46

Unit Learning Outcomes- Unit 3 (Weeks 6-7) ……………………………………………. 47


Big Picture in Focus: ULO-3a…………………………………………………………………..….. 47
Metalanguage…………………………………………………………………………………... 47
Essential Knowledge………………………………………………………………………… 47
Self-Help………………………………………………………………………………………….. 54
In a Nutshell…………………………………………………………………………………….. 56
Q & A List ……………………………………………………………………………………...… 56
Big Picture in Focus: ULO-3b…………………………………………………………………..….. 57
Metalanguage………………………………………………………………………………….. 57
Essential Knowledge………………………………………………………………………… 58
Self-Help………………………………………………………………………………………….. 60
In a Nutshell…………………………………………………………………………………….. 61
Q & A List ………………………………………………………………………………………... 61

Unit Learning Outcomes- Unit 4 (Weeks 8-9) …………………………………………………… 62


Big Picture in Focus: ULO-4a…………………………………………………………………..….. 62
3
College of Engineering Education
2nd Floor, BE Building
Matina Campus, Davao City
Telefax: (082)296-1084
Phone No.: (082)300-5456/300-0647 Local 131
Metalanguage…………………………………………………………………………………... 62
Essential Knowledge………………………………………………………………………… 63
Self-Help………………………………………………………………………………………….. 65
In a Nutshell…………………………………………………………………………………….. 67
Q & A List ……………………………………………………………………………………...… 67
Big Picture in Focus: ULO-4b…………………………………………………………………..….. 68
Metalanguage…………………………………………………………………………………... 68
Essential Knowledge………………………………………………………………………… 68
Self-Help………………………………………………………………………………………….. 73
In a Nutshell…………………………………………………………………………………….. 74
Q & A List ………………………………………………………………………………………... 75

Unit Learning Outcomes- Unit 5 (Weeks 10-11) ………………………………………………. 76


Big Picture in Focus: ULO-5a…………………………………………………………………..….. 76
Metalanguage…………………………………………………………………………………... 76
Essential Knowledge………………………………………………………………………… 76
Self-Help………………………………………………………………………………………….. 83
In a Nutshell…………………………………………………………………………………….. 85
Q & A List ……………………………………………………………………………………...… 85
Big Picture in Focus: ULO-5b…………………………………………………………………..….. 86
Metalanguage………………………………………………………………………………….. 86
Essential Knowledge………………………………………………………………………… 86
Self-Help………………………………………………………………………………………….. 90
In a Nutshell…………………………………………………………………………………….. 91
Q & A List ………………………………………………………………………………………... 91

Unit Learning Outcomes- Unit 6 (Weeks 11-12) ………………………………………………. 92


Big Picture in Focus: ULO-6a…………………………………………………………………..….. 92
Metalanguage…………………………………………………………………………………... 92
Essential Knowledge………………………………………………………………………… 92
Self-Help………………………………………………………………………………………….. 99
In a Nutshell…………………………………………………………………………………….. 101
4
College of Engineering Education
2nd Floor, BE Building
Matina Campus, Davao City
Telefax: (082)296-1084
Phone No.: (082)300-5456/300-0647 Local 131
Q & A List ……………………………………………………………………………………...… 101
Big Picture in Focus: ULO-6b…………………………………………………………………..….. 102
Metalanguage…………………………………………………………………………………... 102
Essential Knowledge………………………………………………………………………… 102
Self-Help………………………………………………………………………………………….. 105
In a Nutshell…………………………………………………………………………………….. 106
Q & A List ………………………………………………………………………………………... 106

Unit Learning Outcomes- Unit 7 (Weeks 13-14) …………………………………………… 107


Big Picture in Focus: ULO-7a…………………………………………………………………..….. 107
Metalanguage…………………………………………………………………………………... 107
Essential Knowledge………………………………………………………………………… 107
Self-Help………………………………………………………………………………………….. 112
In a Nutshell…………………………………………………………………………………….. 113
Q & A List ……………………………………………………………………………………...… 113
Big Picture in Focus: ULO-7b…………………………………………………………………..….. 114
Metalanguage………………………………………………………………………………….. 114
Essential Knowledge………………………………………………………………………… 114
Self-Help………………………………………………………………………………………….. 120
In a Nutshell…………………………………………………………………………………….. 121
Q & A List ………………………………………………………………………………………... 121

Unit Learning Outcomes- Unit 8 (Weeks 15-16) ………………………………………… 122


Big Picture in Focus: ULO-8a…………………………………………………………………..….. 122
Metalanguage…………………………………………………………………………………... 122
Essential Knowledge………………………………………………………………………… 122
Self-Help………………………………………………………………………………………….. 126
In a Nutshell…………………………………………………………………………………….. 128
Q & A List ……………………………………………………………………………………...… 128
Big Picture in Focus: ULO-8b…………………………………………………………………..….. 129
Metalanguage…………………………………………………………………………………... 129
Essential Knowledge………………………………………………………………………… 129
5
College of Engineering Education
2nd Floor, BE Building
Matina Campus, Davao City
Telefax: (082)296-1084
Phone No.: (082)300-5456/300-0647 Local 131
Self-Help………………………………………………………………………………………….. 133
In a Nutshell…………………………………………………………………………………….. 134
Q & A List ………………………………………………………………………………………... 134

6
College of Engineering Education
2nd Floor, BE Building
Matina Campus, Davao City
Telefax: (082)296-1084
Phone No.: (082)300-5456/300-0647 Local 131

Course
Course Outline: CEE 101 –
Coordinator: Differential
Engr. Kimberly Calculus
M. Nepa for Engineering
Email: kimberly.nepa@umindanao.edu.ph
Student Consultation: Online (LMS) or e-mail
Mobile:
Phone:
Effectivity Date: May 2020
Mode of Delivery: Online Blended Delivery
Time Frame: 108 hours
Student Workload: Expected Self-Directed Learning
Pre-requisite: CEE 100
Credit: 5.0 units
Attendance Requirements: For online (virtual/face-to-face) sessions: a
minimum of 95% attendance; for 1-day on-
campus/onsite review: 100% attendance; for 1-day
on-campus/onsite final exam: 100% attendance

Course Outline Policy

Areas of Concern Details


Contact and Non-Contact Hours This 5-unit course self-instructional manual is intended for
blended learning mode of instructional delivery with
scheduled face-to-face or virtual sessions. The expected
number of hours will be 108, including the face-to-face or
virtual sessions which will be arranged by the course
coordinator. The face-to-face sessions shall include the
summative assessment tasks or examinations since this
course is vital in the licensure examination for engineers.

Assessment Task Submission The first assessment task (examination) shall be given on
the 3rd week subsequent to the first day of class. The
remaining assessment tasks shall be handed every after
two (2) weeks of each examination schedule. The
assessment paper shall be attached with a cover page
indicating the name of the course coordinator, date of
submission, and name of the student. The document
should be submitted on the same day through e-mail or
Blackboard LMS. It is also expected that you have already paid
your tuition and other fees before the submission of the
assessment task.
Since this course is included in the licensure examination
for engineers, you will be required to take the Multiple-
Choice Question exam inside the University as your final
exam. This should be scheduled ahead of time by your
course coordinator. This is non-negotiable for all licensure-
based programs.

Penalties for Late The score for an assessment item submitted after the

7
College of Engineering Education
2nd Floor, BE Building
Matina Campus, Davao City
Telefax: (082)296-1084
Phone No.: (082)300-5456/300-0647 Local 131
Assignments/Assessments designated time on the due date, without an approved
extension of time, will be reduced by 5% of the possible
maximum score for that assessment item for each day or
part day that the assessment item is late.

Return of Assignments/ Assessment tasks will be returned to you two (2) weeks
Assessments after the submission with a mark and feedback. This will
be returned by email or via Blackboard portal.
For group assessment tasks, the course coordinator will
require some or few of the students for online or virtual
sessions to ask clarificatory questions to validate the
originality of the assessment task submitted and to ensure
that all the group members are involved.

Assignment Resubmission You should request in writing addressed to the course


coordinator your intention to resubmit an assessment task.
The resubmission is premised on the student’s failure to
comply reasonable circumstances e.g. illness, accidents
financial constraints.

Re-marking of Assessment Papers You should request in writing addressed to the program
and Appeal coordinator your intention to appeal or contest the score
given to an assessment task. The letter should explicitly
explain the reasons/points to contest the grade. The
program coordinator shall communicate with the students
on the approval and disapproval of the request.
If disapproved by the course coordinator, you can elevate
your case to the program head or the dean with the
original letter of request. The final decision will come from
the dean of the college.

Grading System All culled from BlackBoard sessions and traditional


contact:
Exams 1-7 – 30%
Final Exam – 30%
Class Participation:
Quizzes – 15%
Assignment/Plates – 15%
Seatwork/Boardwork/Oral Recitation – 10%
Submission of the final grades shall follow the usual
University system and procedures.

Student Communication You are required to have your own umindanao email
account which is a requirement to access the
BlackBoard portal. Then, the course coordinator shall
enroll the students to have access to the materials and
resources of the course. All communication formats:
chat, submission of assessment tasks, requests etc.
shall be through the portal and other university
recognized platforms.

You can also meet the course coordinator virtually


through the scheduled face-to-face sessions to raise
your issues and concerns.

8
College of Engineering Education
2nd Floor, BE Building
Matina Campus, Davao City
Telefax: (082)296-1084
Phone No.: (082)300-5456/300-0647 Local 131
For students who have not created their student email,
please contact the course coordinator or program head.

Contact Details of the Dean Dr. Charlito L. Cañesares, PME


Email: clcanesares@umindanao.edu.ph
Phone: (082) 296-1084 local 133

Contact Details of the Program Head Engr. Randy E. Angelia, MEP-ECE, MSCpE
Email: randy_angelia@umindanao.edu.ph
Phone: (082) 296-1084 local 133

Students with Special Needs Students with special needs shall communicate with the
course coordinator about the nature of his/her special
needs. Depending on the nature of the need, the course
coordinator with the approval of the program head may
provide alternative assessment tasks or extension of the
deadline of submission of assessment tasks. However,
the alternative assessment tasks should still be in the
service of achieving the desired course learning
outcomes.

Help Desk Contact CEE Blackboard Administrator:


Engr. Jetron A. Adtoon, MSCpE
Email: jadtoon@umindanao.edu.ph
Phone: +63 9055 267834

CEE:
Frida Santa O. Dagatan
Email: cee@umindanao.edu.ph
Phone: +63 9055 267834

GSTC:
Ronadora E. Deala, RPsy, RPm, RGC, LPT
Email: ronadora_deala@umindanao.edu.ph
Phone: +63 921 2122846

Silvino P. Josol
Email: gstcmain@umindanao.edu.ph
Phone: +63 906 0757721

Library Contact Brigida E. Bacani


Email: library@umindanao.edu.ph
Phone: +63 951 3766681

Course Information – see/download course syllabus in the Black Board LMS


9
College of Engineering Education
2nd Floor, BE Building
Matina Campus, Davao City
Telefax: (082)296-1084
Phone No.: (082)300-5456/300-0647 Local 131

CC’s Voice: Hello forthcoming engineers! Welcome to the course CEE 101: Differential
Calculus for Engineering. You are now entering the world of Engineering
Calculus where you need to explore and understand a certain thing by looking
at its small pieces. By this time, I am positive that you really wanted to pursue
your career in the field of engineering and that you have envisioned yourself
inventing, designing, analyzing, building, and testing machines, complex
systems, structures, gadgets and materials for the benefit of humankind.

CO Before going to the next stage of Engineering Calculus, which is the Integral
Calculus or Anti-Differentiation, you have to engage yourself first with
differentiating algebraic and transcendental functions, analyzing and
tracing transcendental curves, and applying the concepts of
differentiation in solving word problems. These are the significant course
outcomes (CO) that you need to demonstrate at the end of this course. Thus,
you are now anticipated to justify the knowledge you have acquired from your
basic mathematics courses in dealing with complex word problems and to read
in advance the concepts of differentiation.

Let us begin!

Big Picture
10
College of Engineering Education
2nd Floor, BE Building
Matina Campus, Davao City
Telefax: (082)296-1084
Phone No.: (082)300-5456/300-0647 Local 131

Week 1-3: Unit Learning Outcomes 1 (ULO1): At the end of the unit, you are expected
to:
a. Evaluate the limits of a function using different techniques;
b. Determine the value/s of domain where the function discontinues and sketch
its graph; and
c. Perform the long method derivation of a function;

Big Picture in Focus:


ULO-1a. Evaluate the limits of a function using different techniques

.
Metalanguage
This section will serve as your word bank where the most essential terms
relevant to the introduction of calculus and ULO-a will be operationally defined to
establish a common frame of reference. You will encounter these terms as we delve
deeper to the study of Differential Calculus. Please refer to these definitions in case you
will find it difficult to understand mathematical concepts in relation with calculus.
1. Domain. The domain of a function is the set of all values that can be plugged
into a function and have the function exist and have a real number for a value.
1.1 It is the set of all possible x-values which will make the function “work”.
1.2 It can be determined by looking for the values of the independent variable
(usually x-values) which can be used in a function.
1.3 When finding for the value/s of domain, it is important to remember that
the denominator of the function cannot be zero and avoid negative values
under the square root sign.
2. Range. The range of a function is simply the set of all possible
values that a function can take.
2.1 It is the resulting y-values we get from substituting all the possible x-values.
2.2 It is the complete set of all possible resulting values of the dependent variable
(usually y-values).
2.3 It is the spread of all possible y-values (minimum y-value to maximum y-
value).
3. Variable. It is a quantity which, during any set of mathematical operations, does
not retain the same value but is capable of assuming different values.
3.1 A variable that represents the “input numbers” for a function is called
independent variables. A variable that represents the “output numbers” is
called dependent variable because its value depends on the value of
independent variable.
4. Constant. It is a quantity which, during any set of mathematical operations,
retains the same value.

11
College of Engineering Education
2nd Floor, BE Building
Matina Campus, Davao City
Telefax: (082)296-1084
Phone No.: (082)300-5456/300-0647 Local 131

5. Polynomial. It can have constants, variables, and exponents, but never division
3x
by a variable (e.g. 5 xy 2−2 x+ 4 y=0, not polynomials: 5 xy−2 and )
x+2
6. Conjugate. It is formed by changing the sign between two terms.

Essential Knowledge
To perform the aforesaid big picture (unit learning outcomes) for the first three (3)
weeks of this course, you need to fully understand the following essential knowledge
that will be laid down in the succeeding pages. Please note that you are not limited to
exclusively refer to these resources. Thus, you are expected to utilize other books,
research articles and other resources that are available in the university’s library (refer to
the Library Contact on page 3).

1. Functions. A function f is a rule that assigns to every number x in a collection D, a


number f (x). The set D is called the domain of the function. And f (x) is called the
value of a function at x , or commonly known as range. The set of ordered pair
(x , f ( x )) is called the graph of f .

Example 1.1: Determine if each of the following are functions.

(a) f ( x )=x 2 +2 This equation is a function. Since no matter what value of


x you put into the equation; it yields exactly one possible
value of f (x).

Let x= 1; Let x=2;


2 2
f ( x )=x +2 f ( x )=x +2
2 2
f ( 1 ) =1 + 2 f ( 2 )=2 + 2
f ( 1 ) =3 f ( 2 )=6

(b) [f ( x ) ]2=x +2 This equation is not a function. At a specific value


of x , there are two (2) possible values of f (x).

Let x= 1; Let x=2;


2
2
[f ( x ) ] =x +2 [f ( x ) ] =x +2
2
2
[f ( 1 ) ] =1+2 [f ( 2 ) ] =2+2
2
2
[f ( 1 ) ] =3 [f ( 2 ) ] =4
f (x)=√ 3 f (x)=√ 4
f ( x )=±1.732 f ( x )=±2

Note:
 “ f ( x ) ” can simply be written as “ y ” or any other function notation like “ g ( x )”, “h ( x )
”, “ R ( x )”, etc.

12
College of Engineering Education
2nd Floor, BE Building
Matina Campus, Davao City
Telefax: (082)296-1084
Phone No.: (082)300-5456/300-0647 Local 131

2. Limits. The limit of a function f (x) as x approaches a is L, can be written as


lim f ( x )=L
x →a

In other words, the value of the function f (x) gets closer and closer to L as x gets
closer and closer to a , without being exactly equal to a . To understand this
concept, let’s have a simple example first.

Example 2.1: Given the function f ( x )=2 x +5 as x approaches 1, find its limit.

To denote this problem, we will write this as:

lim 2 x +5=?
x →1

Let us now look at the function at a point x that gets closer and closer to x=1.

x 0.89 0.99 ₀ 1.1 1.2


f (x) 6.78 6.98 ₀ 7.2 7.4

The table above shows the values of x which are very close to 1 but never equal
to 1. And if we plug in these values to the given function, f (x) moves closer and
closer to 7. Thus, when we evaluate the limit of the given function above, we get
7. It can now be written as:

lim 2 x +5=7
x →1

Please note that the above definition of “Limits” is in its informal form. Let’s stick
with this definition for the meantime since it is easier to understand, and it helps
us to have an idea of just what limits are and its relationship with functions. As we
go further in this course, you will encounter its formal and precise definition.

Why do we have to study limits?


- Because, essentially, that’s what calculus is: the study of limits. Limits
are used to examine function behavior around a specific point, and
without it, it would be very difficult to talk about rate of change.
Therefore, calculus is impossible to do without knowing what to do with
limits.

13
College of Engineering Education
2nd Floor, BE Building
Matina Campus, Davao City
Telefax: (082)296-1084
Phone No.: (082)300-5456/300-0647 Local 131

Before going to evaluate the limits of functions, it would be better if you know
beforehand the “Properties of Limits”. The proof of some these properties can be found
in a separate material (refer to Self-Help).
PROPERTIES OF LIMITS
lim c=c When taking the limit of a constant, the limit is just
x →a ; where a and c are real numbers that same constant.

lim x=a The limit of the function f ( x )=x as x approaches


x →a ; where a and c are real numbers
a given point a is a.
Let a and c be real numbers, let f (x) and g(x ) be defined for all x ’s that lie in some
interval around a.
lim f ( x )=F lim g ( x )=G
x →a x →a

exist with F and G which are real numbers. Thus, holding the following limits: `

lim ( f ( x ) + g ( x )) =F+G The limit of the sum of the functions is the sum of
x →a the limits.

lim ( f ( x )−g ( x ) )=F−G The limit of the difference of the functions is the
x →a
difference of the limits.

lim cf (x )=cF The limit of the product of a constant and a function


x →a
is the product of their limits.

lim ( f ( x ) ∙ g ( x ) )=F ∙ G The limit of the product of the functions is the


x →a
product of the limits.

lim
x →a ( )
f (x) F
= ; where G ≠ 0
g ( x) G
The limit of the quotient of the functions is the
quotient of the limits.
n
lim ( f ( x ) ) =¿ ¿ ;
x →a
The limit of a power is the power of the limit.
where n is a positive integer
1
lim ( f ( x ) ) =¿ ¿ ;
n

x →a
The limit of a power is the power of the limit.
where n is an even number and F> 0, or n is
an odd number and F is any real number
Fa
miliarization of the following properties will be of great help in dealing with limits in the
succeeding examples and activities.
At this point, we are now equipped with enough knowledge to start evaluating limits.
Limits of functions are computed using various techniques such as simple arithmetic,

14
College of Engineering Education
2nd Floor, BE Building
Matina Campus, Davao City
Telefax: (082)296-1084
Phone No.: (082)300-5456/300-0647 Local 131
substitution, or using algebraic simplification. These techniques are illustrated in the
following examples:

2
Example 2.2: Evaluate lim 4 x −1
x →2

2
lim 4 x −1=¿ ¿ ¿ difference of limits
x →2

=¿ product of limits
= 4∙ ¿ limit of constant
= 4 ∙¿ product of limits

= 4 ∙2 ∙ 2−1 = 15 limit of x
2
x −9
Example 2.3: Evaluate lim
x→−3 x +3

2
x 2−9 (−3) −9 0
lim = =
x→−3 x +3 (−3 ) +3 0
0
Substituting −3 for x yields
0
, which is undefined. Let’s find another way to solve this
one. Try algebraic factorization and simplification.
2 lim ( x +3 ) ( x−3 )
x −9 x →−3 factorization
lim =
x→−3 x +3 x +3

¿ lim ( x−3 ) simplification


x→−3

¿ (−3−3 ) limit of x

¿−¿6
x 1

Example 2.4: Evaluate x+ 3 4
lim
x →1 x −1
0
Substituting 1 for x yields
0
, which is undefined. Let us try simplifying the compound
fraction.
x 1 x 1
− −
x+ 3 4 x+3 4 4(x +3) find the LCD of the fraction on
lim =¿ lim ∙ ¿
x →1 x −1 x→ 1 x−1 4(x +3)
the numerator
4 x−(x+ 3)
¿ lim multiply LCD to numerator and denominator
x →1 4 ( x−1 ) ( x+3 )

15
College of Engineering Education
2nd Floor, BE Building
Matina Campus, Davao City
Telefax: (082)296-1084
Phone No.: (082)300-5456/300-0647 Local 131
3 x−3
¿ lim combine similar terms
x →1 4(x−1)(x +3)

3(x−1)
¿ lim factor out the GCF of the numerator (3)
x →1 4(x−1)(x +3)

3
¿ lim cancel out common factor (x-1)
x →1 4(x +3)

3
¿ limit of x
16

x
Example 2.5: Evaluate lim
x →0 √ 1+ x−1

0
Plugging 0 in the numerator and denominator, it yields a value of , which is indeterminate. Since the
0
given function is not a polynomial, we must try another way. There is a lot of ways to simplify the
denominator but there’s a particular method that can eliminate its square root, and it is to multiply it by its
conjugate.

∙√
x x 1+ x +1 conjugate
= multiply by =1
√1+ x−1 √ 1+ x−1 √ 1+ x +1 conjugate

x ( √ 1+ x+ 1)
¿
( √ 1+ x−1)( √ 1+ x +1)

x( √1+ x +1)
¿ 2 note: ( a+ b ) ( a−b )=a2−b 2
( √ 1+ x) −1 ∙ 1

x( √ 1+ x +1)
¿
1+ x−1

x( √ 1+ x +1)
¿ cancel the x
x
¿ √ 1+ x+1

x
Now, we have lim =lim √ 1+ x +1
x →0 √ 1+ x−1 x →0
¿ √ 1+0+1=2

There are times that we need to deal with limits at infinity. Examples are
illustrated:
16
College of Engineering Education
2nd Floor, BE Building
Matina Campus, Davao City
Telefax: (082)296-1084
Phone No.: (082)300-5456/300-0647 Local 131

a
a lim =0
lim r
=0 x→−∞ x
r
x→ ∞ x
where r is a positive rational number and a is any real
where r is a positive rational number and a is any real
number number and x r is defined for x <0

2
2 x +3
Example 2.6: Evaluate lim 2
x→+∞ x −5 x−1


If we plug in + ∞ for the value of x , we get which is indeterminate. To solve

this, we need to factor the largest power of x in the numerator from each term, same
goes with the denominator. This function will become:

3 2
2 2
) x (2+
2 x +3 x
lim 2 =¿ lim
x→+∞ x −5 x−1 x→+∞ 2 5x 1
x (1− 2 − 2 )
x x

3 2
x (2+
2
)
x
= lim 5 1
x→+∞ 2
x (1− − 2 )
X x

3
2+
x2
= lim 5 1
x→+∞
1− −
X x2

3
2+ 2
∞ a
= 5 1
Note:
±∞
=0; where a is any real number
1− −
∞ ∞2

2+ 0
= 1−0−0

=2
2
8x
Example 2.7: Evaluate lim
x→+∞ x +5
17
College of Engineering Education
2nd Floor, BE Building
Matina Campus, Davao City
Telefax: (082)296-1084
Phone No.: (082)300-5456/300-0647 Local 131

This problem has the same concept as the example above. Extract first the
largest power of x in the numerator as well as in the denominator.
2
2 x (8)
8x lim
lim = x→−∞ 5
x→−∞ x+5 x (1+ )
x
8
lim ( x ) ( )
= x→−∞ 1+
5
x
8
(−∞)( )
= 1+
5
−∞

8
( )
= −∞ 1−0 =−∞

Self-Help: You can also refer to the sources below to help you further
understand the lesson.

Feldman, J., & Rechnitzer, A. (2015). Differential Calculus Notes for Mathematics 100.
Lax, P. D., & Terell, M. S. (2014). Calculus with Applications. New York City: Springer.
Terano, H. J. (2015). Calculus 1: A simplified Text in Differential Calculus.

Let’s Check

Activity 1. Since you are now armed with basic knowledge of evaluating limits, it is now your
turn to prove what you have learned. Evaluate the limits of the given functions and show your
solution:
2
1. lim (8−3 x+12 x )
x →2

6+ 4 t
2. lim 2
t →−3 t +1
2
2 x −17 x+ 8
3. lim
x →8 8−x

18
College of Engineering Education
2nd Floor, BE Building
Matina Campus, Davao City
Telefax: (082)296-1084
Phone No.: (082)300-5456/300-0647 Local 131
2
h −4 h−21
4. lim 2
h→ 7 3 h −17 h−28

5. lim
√ 2 y +22−4
y →−3 y +3
2
t +6
6. lim 2
t →0 t −3
2
5 x −16 x +3
7. lim 2
x →3 9−x
3
x +8
8. lim 2
x→−2 x +8 x +12

1 1
+
9. 4 +3 y y
lim
y →−1 y +1

3−t
10. lim
t →3 √ t+ 1−√ 5 t−11

Let’s Analyze
Activity 1. Getting acquainted with the evaluation of limits is not enough. This time, you are
going to evaluate limits of functions again and explain its step-by-step procedure like the
examples from Example 2.2 to Example 2.5.

2
1. lim ( x−4)(x + 8)
x →5

2
2( t−10)
2. lim
t → 10 t

19
College of Engineering Education
2nd Floor, BE Building
Matina Campus, Davao City
Telefax: (082)296-1084
Phone No.: (082)300-5456/300-0647 Local 131
2
5+ x−4 x
3. lim
x→ ∞ √1+ x 2 +2 x 4
1 1
3
+ 2
4. lim 3 t t −1
t→
1 2t−1
2

5. lim
x →1
√ 5 x 3 +4

t−5
6. lim t+ 4
t →−2
( )
lim √ x 2 +8−3
7. x →−1
x+1

x+1
8. lim
x→ ∞
2
√x
2
5+ x−4 x
9. lim
x→ ∞ √1+ x 2 +2 x 4
7 5
3 x + x −15
10. lim 2
x→−∞ 4 x +32 x

20
College of Engineering Education
2nd Floor, BE Building
Matina Campus, Davao City
Telefax: (082)296-1084
Phone No.: (082)300-5456/300-0647 Local 131

In a Nutshell

We are now done with the fundamentals of calculus, which are identifying
functions and computing its limits. Before proceeding to the next unit learning outcomes,
be reminded of some important points when dealing with functions and its limits.

 An equation is said to be a function if it yields only one value of y when it is


computed at a specific value of x .
 The limit is only concerned with what is going on with around a certain point.
 Direct substitution, when evaluating the limits of a function, is a go-to method.
Use other methods or techniques, like factorization and conjugation, only when
this one fails, otherwise you are probably doing more work than you need to be.
 One cannot do Differential Calculus if he/she doesn’t have any prior knowledge
about the evaluation of limits.

Q&A List

If you have any questions regarding functions and limits, kindly write down on the
table provided.

QUESTIONS ANSWERS
1.

2.

3.

4.

5.

21
College of Engineering Education
2nd Floor, BE Building
Matina Campus, Davao City
Telefax: (082)296-1084
Phone No.: (082)300-5456/300-0647 Local 131

Big Picture in Focus:


ULO-1b. Determine the value/s of domain where the function discontinues and sketch
its graph

Metalanguage
You can refer to this section, and the previous one, anytime if you run across
words which you are not familiar with. This will be a common reference in terms of the
definition of mathematical concepts that we are going to discuss.
1. Graph. It is a diagram that shows the relation between variable quantities,
naturally two variables, each measured along one of a pair of axes at right
angles.
2. Rational Function. It is a function that is a ratio of two polynomials. It is called
“rational” because one polynomial is divided by the other like a “ratio”.
3. Quadratic Equation. It is second-order polynomial equation in a single variable
x with a standard form: a x 2 +bx +c=0 where a ≠ 0.
3.1 It always has two solutions based on the fundamental theorem of algebra.
These solutions may be both real or both complex numbers.
4. Quadratic Formula. It is a method to solve for the roots of the quadratic equation.
This method is usually used when the given quadratic equation cannot be
factored out.
4.1 To solve for the roots using this method, it is essential to determine the
values of a , b , and c from the quadratic equation. The quadratic formula is
given by the form:
−b ± √ b2−4 ac
x=
2a

Essential Knowledge
In ULO-b, you will be introduced to the concept of continuity and how it is related
to limits. At the end of this ULO, you are expected to identify if the given functions are
continuous or not and at what point/s it discontinues. There will be graphs included to
deeply understand the concept of continuity.

1. Continuous Function. A function is said to be continuous at a if:


lim f ( x )=f (a)
x →a

With that, we can say that a function, f(x), is continuous at a point a if the limit
exists at a and is equal to f(a). You can also tell that the function is continuous if

22
College of Engineering Education
2nd Floor, BE Building
Matina Campus, Davao City
Telefax: (082)296-1084
Phone No.: (082)300-5456/300-0647 Local 131

you can draw its graph from start to finish without ever once picking up your
pencil/pen, like the graph presented below. The curve is drawn continuously.

But what we are concerned here is to find the value/s of x where the function
discontinues. A function is said to be discontinuous at a if it does not continue at a.
When graphing discontinuous function, you will have to pick up your pencil/pen at least
once to complete the sketch. To have a better understanding with this concept, here are
some graphs to look at:

There are three (3) graphs above representing different sorts of discontinuity. (1)
The first graph with the function, f(x), has a “jump discontinuity” because the function
jumps from a finite value on the left to another value on the right. (2) The graph of g(x) is
what we call the “infinite discontinuity” since the function’s limit is infinity. Lastly, (3) the
function, h(x), has a “removable discontinuity” because we can remove its discontinuity if
it will be redefined at a certain point to make it continuous at that same point.
In our case, we will mostly encounter the function with removable discontinuity
and redefine it to make it continuous. We have already done redefining functions on the
previous ULO, where we need to evaluate the limits of a function that results to
0
numbers with zero (0) as its denominator or . Now, let’s try identifying point/s of
0
discontinuity in a function.

23
College of Engineering Education
2nd Floor, BE Building
Matina Campus, Davao City
Telefax: (082)296-1084
Phone No.: (082)300-5456/300-0647 Local 131

2
x −2 x
Example 1.1: Find the value of the domain where the function, f ( x )= 2 ,
x −4
discontinues.
To solve this, let us graph the function first. Graphing this function requires values for the domain,
x, to be plugged in the given function and to compute the values of its range, f(x). Study the table shown
below.

X -1 0 1 2 3 4 5

f(x) -1 0 1 0 3 2 5
3 0 5 3 7

Based on the table, here is its corresponding graph for reference. This way, it would be easier to
visualize the function’s behavior.

0
As you can see, there is a hole at x=2. Take note that when x=2, the value of f ( x )= which
0
is indeterminate. This means that at this point the function discontinues. So to answer the problem, the
value of the domain where function discontinues is 2.

Let us have another example where we will focus on the function without
sketching a graph.

4 x +10
Example 1.2: Find the value of the domain where the function, f ( x )= 2 ,
x −2 x−15
discontinues.
To easily determine the value/s of x where the function discontinues, all we need to do is to
identify where the denominator is zero since rational functions are continuous everywhere except when it
is divided by zero. To do this, set the denominator equal to zero as what is shown below:

2
x −2 x−15=0
(x +3)(x−5)=0
24
College of Engineering Education
2nd Floor, BE Building
Matina Campus, Davao City
Telefax: (082)296-1084
Phone No.: (082)300-5456/300-0647 Local 131

Thus, the function will not be continuous at x=−3 and x=5 .

2
x −9
Example 1.3: Find the value of the domain where the function, f ( x )= 2 ,
3 x + 2 x−8
discontinues.
Again, all we need do is to isolate first the denominator, equate it to zero, and factor it out. With
this, we will be able to find the values of x where the function is discontinuous.
2
3 x + 2 x−8=0(3 x−4)(x+ 2)=0
3
Therefore, the function will be discontinuous at x= and x=−2.
4

8t
Example 1.4: Find the value of the domain where the function, g ( t )= 2 ,
t −9 t−1
discontinues.
As seen, we have a quadratic equation in the denominator of a rational function. And this
quadratic equation cannot be factored out so to find the value/s of domain where the function
discontinues, we need to use the quadratic formula.

2
t −9 t−1
−(−9)± √ (−9)2−4(1)(−1) 9± √ 85
t= = =−0.10977 ,9.10977
2(1) 2

Self-Help: Refer to the sources below to help you further understand the
lesson.

9 ± √85
Thus, the function will be discontinuous at the points x= .
2

Feldman, J., & Rechnitzer, A. (2015). Differential Calculus Notes for Mathematics 100.
Lax, P. D., & Terell, M. S. (2014). Calculus with Applications. New York City: Springer.
Terano, H. J. (2015). Calculus 1: A simplified Text in Differential Calculus.

Let’s Check
Activity 1. The discussion of the concept of continuity is already done. It is now time for you to
start practicing what you have learned. Determine where the given function is discontinuous.

11−2 x
1. f ( x )= 2
2 x −13 x −7

25
College of Engineering Education
2nd Floor, BE Building
Matina Campus, Davao City
Telefax: (082)296-1084
Phone No.: (082)300-5456/300-0647 Local 131

2
( ) t −1
2. R t = 3 2
t +6 t + t

x−1
3. f ( x )= 2
x + 7 x−4

3
4. G ( z )= 2
2 z +3 z−4

Let’s Analyze
Activity 1. Let’s try another one with a little bit of twist. Determine where the given function is
discontinuous and graph it.

5 x+ 1
1. f ( x )= 2
x +4

3x
2. h ( x )=
x−5

1
3. g ( x )= x
2 −8

2
t +t−15
4. r ( t )=
t−8

2
2 l −7 l
5. g ( l )= 2
l +2 l−5

In a Nutshell
We have already dealt with the concept of continuity by sketching its graph and
identifying the value/s of domain that make/s the function discontinues. Here are some
points you should remember:

 A function is said to be continuous at a if lim f ( x )=f (a).


x →a

26
College of Engineering Education
2nd Floor, BE Building
Matina Campus, Davao City
Telefax: (082)296-1084
Phone No.: (082)300-5456/300-0647 Local 131

 Every polynomial is continuous everywhere. In the same way, every rational


function is continuous except when its denominator is zero.
 Listed below are functions that are continuous everywhere in their domain:
o Polynomials, Rational Functions
o Roots and Powers
o Trigonometric Functions and their inverses
o Exponential and the Logarithm functions

Q&A List

If you have any questions regarding continuity, kindly write down on the table
provided.

QUESTIONS ANSWERS
Big Picture in Focus:
1. ULO-1c. Perform the long method derivation of a function

2.

3.

4.

5.

Metalanguage
If you run across terms you are unfamiliar with, refer to this section. This will
serve as our common reference in dealing with concepts of differential calculus.

27
College of Engineering Education
2nd Floor, BE Building
Matina Campus, Davao City
Telefax: (082)296-1084
Phone No.: (082)300-5456/300-0647 Local 131

1. Rate of Change. It is the rate at which one quantity changes in relation to another
quantity. One best example for this one is the slope of a line where there is a change
in y over the change in x.
2. Rationalization. It is a process to eliminate all the radicals that are either on the
numerator or the denominator since, sometimes, one encounters radical expressions
which are not perfect roots called irrational numbers or expressions.
2.1 To do this method, you need to multiply both the numerator and the denominator
by a radical that will get rid of the radical in either the numerator or the
denominator.

Essential Knowledge
In ULO-c, we are going to define and discuss the concept of rate of change, in
which mathematically speaking, is the derivative. At the end of this section, you need to
demonstrate how to do derivation using the long method.

1. The Derivative. A function, f , is called differentiable at a if the difference


quotient
f ( x+ ∆ x )−f (x)
∆x
approaches to a limit as ∆ x approaches 0. This limit is called the derivative
of f at x and is denoted by f ' (x):

f ( x +∆ x )−f (x )
f ' ( x )= lim
∆ x →0 ∆x

The concept of derivative can, actually, be demonstrated by the slope. Study


the illustrations given below.

Change∈Y Change in Y
Slope=
Change∈ X
Change in X

This formula computes the average


A
slope between the two points, A and B.
14
14
B average slope=
28 27
College of Engineering Education
2nd Floor, BE Building
Matina Campus, Davao City
Telefax: (082)296-1084
Phone No.: (082)300-5456/300-0647 Local 131

27

But how can we calculate for the slope


at a certain point? There’s nothing to That is where derivatives show up. In
measure. derivatives, we use a small difference,
and then have it contract towards zero
to calculate its rate of change.
0
slope= = ???
0 We can now conclude that:
?
Change∈Y Δ y
Slope= =
Change∈ X Δ x

Take note that “∆ ” is a symbol for “delta” which means “change” or a “change in”
and from the given illustration, we can see that:

x transforms from x to ∆ x f (x)


f (x+ ∆ x)
y transforms from f (x) to f (x+ ∆ x)

That is how the derivative and slope are related. To know the rate of change in a
certain point of a curve or a line, we need do derivation since f ' (x) or y ' represents the
rate of change of f ( x ) .

This time let us practice doing the long-method derivation of a function to further
understand this concept. First, study the steps of long-method derivation presented
below and look at the examples following it.

Steps in doing long-method derivation:

Δ y f ( x+ ∆ x )−f (x)
1. Fill in the slope-formula given by: =
Δx ∆x
2. Simplify it.
3. Let ∆ x approach zero (0).

Example 1.1: Find the derivative of f ( x )=x 2.


29
College of Engineering Education
2nd Floor, BE Building
Matina Campus, Davao City
Telefax: (082)296-1084
Phone No.: (082)300-5456/300-0647 Local 131

Let us start with transforming f ( x )=x 2 to f ( x + ∆ x ) =( x +∆ x )2.


Expand ( x +∆ x )2: f ( x + ∆ x ) =x2 +2 x ∆ x+¿
f ( x+ ∆ x )−f (x)
Use the slope formula:
∆x

Plug in the equivalent values of f ( x + ∆ x ) and f ( x ) :


[ x 2+2 x ∆ x+ ( ∆ x )2 ]−x 2
∆x
2
2 x ∆ x +( ∆ x )
Simplify by combining similar terms:
∆x

Simplify further by dividing the numerator by ∆ x : 2 x+ ∆ x


Lastly, as ∆ x approaches 0, we get: 2 x
Thus, the derivative of x 2 is 2 x .

We can use some notations to simplify the symbols needed in derivation. Let us
use dx instead of “∆ x approaches 0” and “the derivative of” can be written as
{d} over {dx} . So now we can write:
d 2
( x )=2 x
dx

Note: oftentimes, f ' (x) is used also for “the derivative of”.

Let’s try more examples using the notations discussed.

Example 1.2: Solve for f ' (x) the function, f ( x )=2 x 2−16 x +35.

' f ( x +∆ x )−f (x )
f ( x )= lim
∆ x →0 ∆x
2 2
2(x +∆ x ) −16 ( x+ ∆ x )+ 35−(2 x −16 x +35)
f ' ( x )= lim
∆ x →0 ∆x

' 2 [ x 2+2 x ∆ x+ ( ∆ x )2 ]−16 x−16 ∆ x +35−2 x 2 +16 x−35


f ( x )= lim
∆ x →0 ∆x
2 2 2
2 x + 4 x ∆ x+2 ( ∆ x ) −16 x−16 ∆ x+ 35−2 x +16 x−35
f ' ( x )= lim
∆ x →0 ∆x

30
College of Engineering Education
2nd Floor, BE Building
Matina Campus, Davao City
Telefax: (082)296-1084
Phone No.: (082)300-5456/300-0647 Local 131

2
' 4 x ∆ x +2 ( ∆ x ) −16 ∆ x
f ( x )= lim
∆ x →0 ∆x

∆ x (4 x+ 2 ∆ x−16)
f ' ( x )= lim
∆ x →0 ∆x
'
f ( x )= lim 4 x +2 ∆ x−16
∆ x →0

'
f ( x )=4 x+ 2 ( 0 )−16
'
f ( x )=4 x−16

x
Example 1.3: Solve for f ' (x) the function, f ( x )= .
x +1

f ( x +∆ x )−f (x )
f ' ( x )= lim
∆ x →0 ∆x

f ' ( x )= lim
∆ x →0
( ∆1x )( [ x+x +∆∆x x]+1 − x +1x )
f ' ( x )= lim
∆ x →0
( )(
1
∆x
[ x + ∆ x ][ x+1 ]−x [x +∆ x +1]
[ x+ ∆ x+1 ] [x +1] )
f ' ( x )= lim
∆ x →0
( ∆1x ) ¿ ¿
' 1
f ( x )= lim
∆ x →0 ( x + ∆ x+1)(x +1)

' 1
f ( x )=
(x +0+1)(x +1)

' 1
f ( x )=
(x +1)(x+ 1)

' 1
f ( x )= 2
(x +1)
31
College of Engineering Education
2nd Floor, BE Building
Matina Campus, Davao City
Telefax: (082)296-1084
Phone No.: (082)300-5456/300-0647 Local 131

Example 1.4: Solve for f ' (x) the function, f ( x )= √ 5 x−8 .

' f ( x +∆ x )−f (x )
f ( x )= lim
∆ x →0 ∆x

f ' ( x )= lim
√ 5 ( x + ∆ x )−8− √5 x−8
∆ x →0 ∆x

It looks like we are stuck here. Let us now recall the concept of rationalization (refer to the
Metalanguage) since this is what we are going to do with this problem. Probably in your previous
mathematics subject you have just tried rationalizing the denominator but in our example, we will
rationalize the numerator.

f ' ( x )= lim
√ 5 ( x + ∆ x )−8− √5 x−8 ∙ √ 5 ( x + ∆ x ) −8+ √ 5 x−8
∆ x →0 ∆x √ 5 ( x + ∆ x ) −8+ √ 5 x−8
5 ( x +∆ x )−8−( 5 x−8 )
f ' ( x )= lim
∆ x →0 ∆ x ( √ 5 ( x + ∆ x )−8+ √ 5 x−8 )

' 5 x +5 ∆ x−8−5 x+ 8
f ( x )= lim
∆ x →0 ∆ x ( √ 5 ( x + ∆ x )−8+ √ 5 x−8 )

' 5∆ x
f ( x )= lim
∆ x →0 ∆ x ( √ 5 ( x + ∆ x )−8+ √ 5 x−8 )

' 5
f ( x )= lim
∆ x →0 ( √5 ( x+ ∆ x )−8+ √5 x−8 )
' 5
f ( x )=
( √ 5 ( x+ 0 )−8+ √5 x−8 )
' 5
f ( x )=
( √ 5 x−8+ √ 5 x −8 )

' 5
f ( x )=
2 √ 5 x−8

In dealing with the long-method derivation, you still need to recall some of the
algebraic concepts you have encountered just like performing rationalization. The

32
College of Engineering Education
2nd Floor, BE Building
Matina Campus, Davao City
Telefax: (082)296-1084
Phone No.: (082)300-5456/300-0647 Local 131

success in doing the long-method derivation is still up to you and the process you are
going to do.

Let’s Check

Activity 1. At this point, I am positive that you will be ready to perform long-method derivation.
To prove this, find the derivative of the following functions using the long method.

1. f ( x )=14

2. G ( t ) =2−27 t

3. h ( x )=5 x 3−1

4
4. R ( z )=
z

5. k ( x )=x 3−x 2 +2 x−6

Let’s Analyze

Activity 1. You are improving. Now, let us try doing the long-method derivation again but this
time, you need to explain the process you did.

x +3
1. f ( x )=
x +7

2. r ( t )=5 t 2−t+9
3. g ( z )=√ 3 z −2
4 m−15
4. k ( m )=
3m

5. h ( x )= x 2−5 x

Self-Help: You can also refer to the sources below to help you further
understand the lesson.

33
College of Engineering Education
2nd Floor, BE Building
Matina Campus, Davao City
Telefax: (082)296-1084
Phone No.: (082)300-5456/300-0647 Local 131

Lax, P. D., & Terell, M. S. (2014). Calculus with Applications. New York City: Springer.
Terano, H. J. (2015). Calculus 1: A simplified Text in Differential Calculus.

In a Nutshell
The discussion of the introduction for derivatives is done. Here are some points
to consider in performing long-method derivation.

 The process of calculating the derivative of a function is called “differentiation”.


 To get a derivative, you need to do differentiation.
 The formula used in performing the long-method differentiation comes from the
slope formula. That is why when doing it, you just need to fill in the formula with
appropriate values, simplify it as best as you can, and let ∆ x approach zero.

Q&A List

If you have any questions regarding continuity, kindly write down on the table
provided.

QUESTIONS ANSWERS
1.

2.

3.

4.

5.

34
College of Engineering Education
2nd Floor, BE Building
Matina Campus, Davao City
Telefax: (082)296-1084
Phone No.: (082)300-5456/300-0647 Local 131

Big Picture in Focus:


ULO-2a. Apply the Differentiation Rules of Algebraic Function

Big Picture

Week 4-5: Unit Learning Outcomes 2 (ULO2): At the end of the unit, you are
expected to:
a. Apply the Differentiation Rules of Algebraic Function; and
b. Solve for the nth order derivative of a function—either implicit or explicit.

Metalanguage

35
College of Engineering Education
2nd Floor, BE Building
Matina Campus, Davao City
Telefax: (082)296-1084
Phone No.: (082)300-5456/300-0647 Local 131

This section will be your reference regarding some terms related to the rules of
differentiation of algebraic functions. This will serve as or common reference in dealing
with terms we are unfamiliar with or the words we are not quite sure of its meaning.

1. Auxiliary Variable. It is a variable found in the main function but is not


necessarily part of the solution of a function. In differential calculus, it is a
variable that has an equivalent function in terms of the independent variable. This
can be usually found in the application of chain rule.

Essential Knowledge
In ULO-a, you will run across numerous rules of differentiation which you need to
familiarize since this will be great for saving time in finding derivatives of functions. You
are already acquainted with the long-method differentiation in the previous ULO, this
time, you will be accustomed with the most efficient way of differentiating functions and
you will clearly see the difference between the two methods.

1. Differentiation of Algebraic Functions. Mathematicians developed a way to avoid


doing the long and time-consuming process of differentiation. This established
method will be essential in finding the derivatives of complex functions. The standard
formulas and properties that will save you time in dealing with derivatives are
presented in the next page:

36
College of Engineering Education
2nd Floor, BE Building
Matina Campus, Davao City
Telefax: (082)296-1084
Phone No.: (082)300-5456/300-0647 Local 131

DIFFERENTIATION RULES FOR ALGEBRAIC FUNCTIONS


d
c=0 ; where c is constant The derivative of a constant is zero (0).
dx

If u and v are functions, then the following formulas are true by the definition of
derivative:
d du dv The derivative of the sum of two functions is equal
( u+ v )= +
dx dx dx to the sum of their derivatives.
d dv du The derivative of the product of two functions is
( uv )=u + v
dx dx dx equal to the product of the first function and the
derivative of the second plus the product of the
second function and the derivative of the first.

The derivative of the quotient of two functions is


du dv
−u
v equal to the product of the denominator and the
d u
dx v()
=
dx
v
2
dx derivative of the numerator minus the product of
the numerator and the derivative of the
denominator, divided by the square of the
denominator.
d n n−1
(x )=n x ; The derivative of a variable with a positive integer
dx
exponent is the product of the exponent and the
wheren is a positive integer variable raised to the integer exponent minus one.

d n n−1 du
(u )=n u ; The derivative of a function with a power is the
dx dx product of the power, the function raised to its
whereu is a function and n is a positive power minus one, and the derivative of the
function. This is called the general power formula.
integer
du
d dx 1
( √ u)= ; This is a special case where n= . Its derivative
dx 2 √u 2
is the derivative of the function divided by two times
1
whereu is a function and n= the square root of the function.
2
Let us now have examples of each of the rules for you to appreciate the method more.
The proof of the standard formulas presented above will be on separate section. You
can also prove it on your own using the long-method differentiation.

Example 1.1: Differentiate the following functions:

(a) y=25
y '=0
 When differentiating a constant, it is always equal to zero (0).

37
College of Engineering Education
2nd Floor, BE Building
Matina Campus, Davao City
Telefax: (082)296-1084
Phone No.: (082)300-5456/300-0647 Local 131

(b) f ( x )=8 x −3

[ dx dx ]
f ' ( x )= 8 ( d
x )+ x ( 8 ) −0
d

f ' (x)=8 ( 1 )+ x(0)

f ' (x)=8

 The given function has a product of a constant and a variable where we need to
use the product formula for differentiation.

(c) f ( x )=x 2 +6 x−1

f ' ( x )=2 x 2−1+ 6 ( dxd x )−0


f ' ( x )=2 x +6

 In this example, we used the differentiation for sum, power rule, and the product
rule.

x
(d) y= 2
x −1

( x 2−1 ) ( 1 )−(x)(2 x )
y '=
¿¿
2 2
x −1−2 x
y '=
¿¿
2
' −1−x
y=
¿¿

 Remember the quotient rule of differentiation, it is applied here.

(e) y=( 3 x 3 +2 ) ( x 2−1 )


d 2
y =( 3 x +2 )
' 3
( x −1 ) + ( x 2−1 ) d ( 3 x3 +2 )
dx dx

y ' =( 3 x 3 +2 ) ( 2 x ) + ( x 2−1 ) (9 x2 )
' 4 4 2
y =6 x + 4 x +9 x −9 x

38
College of Engineering Education
2nd Floor, BE Building
Matina Campus, Davao City
Telefax: (082)296-1084
Phone No.: (082)300-5456/300-0647 Local 131

' 4 2
y =15 x −9 x + 4 x
 This example shows how the differentiation of the product of two functions work.

(f) y=(x 2−4 x+ 8)3


3−1 d 2
y =3 ( x −4 x+ 8 )
' 2
(x −4 x +8)
dx
2
y =3 ( x −4 x+ 8 ) (2 x−4 )
' 2

2
y =( x −4 x +8 ) (6 x−12)
' 2

 This is an example of the general power formula. Take note that you cannot multiply the
two functions together since the first one has a power of 2 and the other one has a power
of one 1. If you want to simplify the answer further, you may expand the first function by
multiplying it by itself and multiplying it, afterwards, with the second function.

There is also a special case called the Chain Rule where there is an auxiliary variable
in the primary function. This auxiliary variable is the function of the independent variable. This
simply means that this auxiliary variable has an equivalent function in terms of the independent
variable. The Chain Rule is given by the form:
dy dy du
= ∙
dx du dx

where y=f (u) and u=∅ (x)


An example is illustrated below to fully comprehend the idea of chain rule.

Example 1.2: Derive y=3 u 2+1 if u=√ x−1

dy d d
= ( 3 u +1 ) ∙ ( √ x−1)
2
use Chain Rule
dx du dx

1
dy 1 −1

dx
=6 u ∙ (x−1) 2 (1)
2
differentiate √ x−1 using general power rule

dy 1
=6( √ x−1)∙ substitute the equivalent value of u
dx 2 √ x−1

dy 6
= =3 cancel √ x−1 and simplify
dx 2

Self-Help: Refer to the sources below


39 to help you further understand the
lesson.
College of Engineering Education
2nd Floor, BE Building
Matina Campus, Davao City
Telefax: (082)296-1084
Phone No.: (082)300-5456/300-0647 Local 131

Feldman, J., & Rechnitzer, A. (2015). Differential Calculus Notes for Mathematics 100.

Let’s Check

Activity 1. Find the derivative of the following functions using the rules presented on the
previous pages. Box your final answer.

1. f ( x )=x 3 +12 x 2−7 x+10

2. G ( t ) =4 t 2 (t−1)3
3
x −1
3. y=
x+2

4z
4. h ( z )= 3
z

5. y=¿

Let’s Analyze

Activity 1. Solve or the derivative of the following and label each step of the process.

1. f ( x )= ( x +1 )2 ( x−1 ) ( x2 +1 )

( t+3 )3
2. R ( t )=
t 2−3
3. h ( z )=√ z 2+ 4


3
m +2 m
4. l ( m )= 2
7m

( )
2 2
x +3 x +9
5. y=
x−2

Activity 2. The following functions are similar from the long-method differentiation activity. Since
you were done doing its long-method derivation, it’s now time for you to prove if your answers in
long-method differentiation are the same if you use the rules of differentiation.

x +3
1. f ( x )=
x +7

40
College of Engineering Education
2nd Floor, BE Building
Matina Campus, Davao City
Telefax: (082)296-1084
Phone No.: (082)300-5456/300-0647 Local 131

2. r ( t )=5 t 2−t+9

3. g ( z )=√ 3 z −2
4 m−15
4. k ( m )=
3m

5. h ( x )= x 2−5 x

In a Nutshell
These are some points to remember if you are going to use the rules of
differentiation for algebraic functions:

 The derivative of a function using the rules of differentiation is similar to the


derivative of the same function using the long-method derivation.
 You need to familiarize the rules of differentiation for algebraic function to save a
great time in solving for the derivatives of functions instead of utilizing the long
method.
 The standard formulas for differentiation will be your foundation in dealing with
the advanced type of functions.

Q&A List

If you have any questions regarding continuity, kindly write down on the table
provided.

QUESTIONS ANSWERS
1.

2.

3.

4.

5.

41
College of Engineering Education
2nd Floor, BE Building
Matina Campus, Davao City
Telefax: (082)296-1084
Phone No.: (082)300-5456/300-0647 Local 131

Big Picture in Focus:


ULO-2b. Solve for the nth order derivative of a function
--- either implicit or explicit

Metalanguage
In this section, you will find terms that are related to the differentiation of
functions. These terms will be your basis as to what they are used for or how they are
being used in this concept. You may go back here anytime if you run across a certain
term you find ambiguous.
1. Implicit Function. It is a function that is usually presented in terms of both
independent and dependent variables (e.g. x 2+ y 2=1)
2. Explicit Function. It is a function that is given in terms of the independent
variable only (e.g. y= √ 1− x2)

Essential Knowledge
In ULO-e, you are expected to derive functions correctly in its nth order whether
it is implicit or explicit. Equipped with the knowledge of differentiating algebraic functions
according to certain formulas or rules, you can achieve the intended learning outcome.
These are the things you need to know in addition to the rules of differentiation for
algebraic functions.

1. Higher Order Derivatives. This refers to any derivative beyond the first
derivative. In solving higher order derivatives or the nth order derivatives, you
need to repeatedly differentiate the given function. The idea of this is
presented by the examples below:

Example 1.1: Find the 2nd derivative of f ( x )=2 x 3−3 x 2+ 5 x −8

To find the second (2nd) derivative of the given function, you need to first determine its
first (1 ) derivative. To denote the first derivate of f ( x ) , we write f ' ( x ) therefore f ' ' ( x ) , read as “
st

f double prime x ”, will be written for the 2nd derivative.


2
f ' ( x )=6 x −6 x+5 first derivative
42
College of Engineering Education
2nd Floor, BE Building
Matina Campus, Davao City
Telefax: (082)296-1084
Phone No.: (082)300-5456/300-0647 Local 131

f ' ' ( x ) =12 x−6 x second derivative

There are other notations used for representing the higher order derivatives.
Study the given table and familiarize the following:

2nd derivative 3rd derivative 4th derivative nth derivative


(4 ) (n)
most common notation f ' ' (x) f ' ' ' ( x) f (x) f (x )
2 3 4 n
Leibniz notation
d f d f d f d f
2 3 4 n
dx dx dx dx
2 3 4 n
alternative form of d d d d
Leibniz notation 2
[ f ( x )] 3
[ f ( x )] 4
[f ( x ) ] n
[f ( x ) ]
dx dx dx dx
2 3 4 n
Euler’s notation D f D f D f D f

Let us have another set of examples. This time we will use variety of notations.

Example 1.2: Find the 3rd derivative of the following functions:

(a) y=4 x 5 +6 x 3+ 2 x +1

4 2
y '=20 x + 18 x +2
3
y ' '=80 x +36 x
2
y ' ' '=240 x +36

1 2
(b) f ( x )= 2
−7 x
( x +2 )

−2 2
1 −2
f ( x )=( x +2) −7 x transform to ( x +2) to use general power formula
( x+2 )2
−2−1
f ' ( x )= (−2 )( x +2 ) (1)−14 x
−3
f ' ( x )= (−2 )( x +2 ) −14
−3−1
f ' ' ( x ) =( 6 ) ( x+2 ) (1)−0
−4
f ' ' ( x ) =( 6 ) ( x+2 )
−4 −1
f ' ' ' ( x)=(−24 )( x +2 ) (1)
'' ' −5
f (x)= (−24 ) ( x+ 2 )
43
College of Engineering Education
2nd Floor, BE Building
Matina Campus, Davao City
Telefax: (082)296-1084
Phone No.: (082)300-5456/300-0647 Local 131

'' ' −24


f (x)=
( x +2 )5

The functions presented in our examples are explicit functions, these are usually the
functions we have dealt with already. Now, let us try differentiating functions in an implicit form.
Differentiating functions in this form is easier when not transformed into an explicit one so we
will be differentiating these functions as it is.

Example 1.3: Differentiate the following implicit functions:

(a) x 2−xy + y 2 + x=1

d 2 2 d
(x −xy + y + x )= (1)
dx dx
d 2 d d d d
(x )− (xy )+ ( y¿ ¿ 2)+ ( x)= (1)¿
dx dx dx dx dx

dy dy
2 x−x − y +2 y +1=0
dx dx

dy dy
−x +2 y =−1−2 x + y
dx dx

dy
(−x +2 y )=−1−2 x + y
dx

dy −1−2 x + y
=
dx −x +2 y

x
(b) 3
=0
y
−3 3
x y =0 Transfer y to the numerator and make its exponent
positive

−3
x (−3)( y ¿¿−3−1) ( dy ) + y ( dx)=0 ¿ Product Rule of Differentiation

−4 −3
−3 x y dy + y dx=0
−4 −3
−3 x y dy=− y dx

44
College of Engineering Education
2nd Floor, BE Building
Matina Campus, Davao City
Telefax: (082)296-1084
Phone No.: (082)300-5456/300-0647 Local 131

−3
' −y dy
y= =y'
−3 x y
−4
dx
' y
y=
3x

(c) ( x + y )2=3 xy
2−1
2 ( x+ y ) (dx+ dy)=3(xdy + ydx )
2(x + y )(dx +dy )=3 xdy +3 ydx
2(xdx + xdy + ydx + ydy )=3 xdy +3 ydx
2 xdx+ 2 xdy +2 ydx +2 ydy=3 xdy +3 ydx
2 xdy +2 ydy−3 xdy=3 ydx−2 ydx−2 xdx
2 ydy−xdy = ydx−2 xdx
( 2 y−x ) dy=( y−2 x)dx
dy ( y−2 x)
=
dx ( 2 y−x )

Example 1.4: Find the 2nd derivative of the given implicit functions:

(a) x 3 + xy=9 y
3−1
3x dx + xdy + ydx=9 dy
2
3 x dx + xdy + ydx =9 dy
2
3 x dx + ydx =9 dy−xdy
dy (9−x )=dx ( 3 x2 + y )
2
dy 3 x + y
=
dx 9−x
2
d 2 y ( 9−x )( 6 x +0 ) −(3 x + y)(−1)
=
d x2 (9−x)2
2 2
d 2 y (54 x−6 x )−(−3 x − y)
=
d x2 ( 9−x )2
2
d y
=54 x −6 x +3 x + y ¿ ¿ 2
2 2
2
dx ( 9−x )

45
College of Engineering Education
2nd Floor, BE Building
Matina Campus, Davao City
Telefax: (082)296-1084
Phone No.: (082)300-5456/300-0647 Local 131

2 2 2
d y 54 x−3 x + y −3 x −54 x− y
2
= 2
= 2
dx ( 9−x ) (9−x)

2
x
(b) y 2=
x+ y
2 2
y ( x+ y )=x
2 3 2
xy + y =x
[ x ( 2 ydy ) + y ¿¿ 2 dx ]+3 y 2 dy =2 xdx ¿
2 2
2 xydy + y dx+3 y dy=2 xdx
( 2 xy +3 y 2 ) dy=( 2 x− y 2 ) dx
2
dy 2 x− y
=
dx 2 xy +3 y 2
d y ( 2 xy+ 3 y ) ( 2 ) −(2 x− y )(2 y )
2 2 2

2
= 2 2
dx (2 xy +3 y )
d y ( 4 xy+ 6 y )−(4 xy −2 y )
2 2 3

2
= 2 2
dx (2 xy +3 y )
2 2 3
d y 4 xy + 6 y −4 xy +2 y
2
= 2 2
dx (2 xy + 3 y )
2
d2 y 2 y (3+ y)
=
d x [( y)(2 x +3 y)]2
2

2
d2 y 2 y (3+ y ) 2(3+ y )
= 2 =
d x ( y) (2 x+3 y ) (2 x +3 y )2
2 2

Self-Help: Refer to the sources below to help you further understand the
lesson.

Terano, H. J. (2015). Calculus 1: A simplified Text in Differential Calculus.

Let’s Check

Activity 1. Find the 2nd derivative of the following functions:

1. y=x 2−7 x +1
46
College of Engineering Education
2nd Floor, BE Building
Matina Campus, Davao City
Telefax: (082)296-1084
Phone No.: (082)300-5456/300-0647 Local 131

2. x=(2−3 t)2

3. y= √ 16+ x 2

Activity 2. Find the 3rd derivative of the following functions:

1. y=3 x 4 −x 3+5 x 2 +4 x+ 9

2. x=t 2+ √ t +2 t

3. z=( y−3)( y +5)3

dy
Activity 3. Find of the following implicit functions:
dx

1. x 2+ 2 y 2 =4 y

2. 3 y 2 ( x + y ) =x− y
1 1
3. x 2 −x y 2 =− y 2

2
d y
Activity 4. Find 2 of the following implicit functions:
dx

1. x 3−9 y=xy

2. y 2−3 y 2 √ x=xy

3. √ x+ y= y 2

Let’s Analyze

Activity 1. Find the 2nd derivative of the even-numbered functions and the 3 rd derivative of the
odd-numbered ones:

1. f ( x )=(x +2)4

x 2+ 2
2. y=
√x
47
College of Engineering Education
2nd Floor, BE Building
Matina Campus, Davao City
Telefax: (082)296-1084
Phone No.: (082)300-5456/300-0647 Local 131

3. h ( z )=z 6 +3 z 5−4 z 4 + z 3+2 z 2 +9 z+ 10

4. f ( x )= √
x 2−3
x

5. x=(2t +1)3

2
6. y= 2 4
(6+2 x−x )

1
7. g ( h )=4 √ h −
5
− √h
3
2
8h

1
8. x=
√ 6 t+t 4
9. y=6 x −2+7 x −3 −x−4

32 1
10. f ( x )= √ x − 4 +
3 2

√ x 3 √ x5

Activity 2. Find (a) y ' by equating the function to y and differentiating it; (b) y ' by implicit
differentiation; and (c) y ' ' using either of the solved y ' .

1. 8 xy +2 x 4 y−3=x 3

2. x 2= y 2− √ x 3 +2 y
2 2
( x + y)
3. 3 x 2=
y
4. (x +2 y)2=4 xy + √ x

5. x 3− y 3=3

In a Nutshell
You need to take note of these points when you run across nth order derivative
of both explicit and implicit functions:
48
College of Engineering Education
2nd Floor, BE Building
Matina Campus, Davao City
Telefax: (082)296-1084
Phone No.: (082)300-5456/300-0647 Local 131

 To perform higher order differentiation, you just need to do derivation after


another derivation.
 In differentiation implicit functions, it would be easier to derive it in its original
form than transforming it into explicit function.
 Most importantly, there are many ways to find the derivative of a function. All you
have to do is find the simplest method.

Q&A List

If you have any questions regarding continuity, kindly write down on the table
provided.

QUESTIONS ANSWERS
1.

2.

3.

4.

5.

Big Picture
Week 6-7: Unit Learning Outcomes 3 (ULO 3): At the end of the unit, you are
expected to:

49
College of Engineering Education
2nd Floor, BE Building
Matina Campus, Davao City
Telefax: (082)296-1084
Phone No.: (082)300-5456/300-0647 Local 131

a. Calculate the tangents and normal to plane curves; and


b. Determine increasing and decreasing functions and its local maximum and
minimum

Big Picture in Focus:


ULO-3a. Calculate the tangents and normal to plane curves.

Metalanguage
This section will serve as your word bank where the most essential terms
relevant to the differential calculus and ULO-a will be operationally defined to establish
a common frame of reference. You will encounter these terms as we delve deeper to
the study of Differential Calculus. Please refer to these definitions in case you will find it
difficult to understand mathematical concepts in relation with calculus. Also, you need
the skill in deriving to continue to the next pages.

We often need to find tangents and normal to curves when we are analyzing
forces acting on a moving body.
1. A tangent to a curve is a line that touches the curve at one point and has the
same slope as the curve at that point.
2. A normal to a curve is a line perpendicular to a tangent to the
curve..

50
College of Engineering Education
2nd Floor, BE Building
Matina Campus, Davao City
Telefax: (082)296-1084
Phone No.: (082)300-5456/300-0647 Local 131

Essential Knowledge
To perform the aforesaid big picture (unit learning outcomes), you need to fully
understand the following essential knowledge that will be laid down in the succeeding
pages. Please note that you are not limited to exclusively refer to these resources.
Thus, you are expected to utilize other books, research articles and other resources that
are available in the university’s library (refer to the Library Contact on page 3).

This unit explains how differentiation can be used to calculate the equations of
the tangent and normal to a curve. The tangent is a straight line which just touches the
curve at a given point. The normal is a straight line which is perpendicular to the
tangent.
To calculate the equations of these lines we shall make use of the fact that the
equation of a straight line passing through the point with coordinates (x1,y1) and having
slope m is given by

We also make use of the fact that if two lines with slope m 1 and m2 respectively
are perpendicular, then m1m2 = −1. To master the techniques explained here it is vital
that you undertake plenty of practice exercises so that they become second nature.
After reading this text, and/or viewing the video tutorial on this topic, you should be able
to:
• calculate the equation of the tangent to a curve at a given point
• calculate the equation of the normal to a curve at a given point

Consider a function f(x) such as that shown in Figure 1. When we calculate the
derivative, f′, of the function at a point x = a say, we are finding the slope of the tangent
to the graph of that function at that point. Figure 1 shows the tangent drawn at x = a.
The slope of this tangent is f′(a).

Figure 1. The tangent drawn at x = a has slope f′(a).

We will use this information to calculate the equation of the tangent to a curve at a
particular point, and then the equation of the normal to a curve at a point.

51
College of Engineering Education
2nd Floor, BE Building
Matina Campus, Davao City
Telefax: (082)296-1084
Phone No.: (082)300-5456/300-0647 Local 131

Calculating the equation of a tangent


Example 1: Find the equation of the tangent to the curve y=3x2 at the point (1, 3).
Sketch the curve and the tangent.
Step 1: Find the derivative
Use the rules of differentiation:
y = 3x2
y’ = 2(3x) = 6x
Step 2: Calculate the gradient/slope of the tangent
To determine the slope of the tangent at the point (1, 3), we substitute the
x-value into the equation for the derivative.
y’ = 6x = 6(1) = 6
Step 3: Determine the equation of the tangent
y – y1 = m(x – x1)
y – 3 = 6(x – 1)
y = 6x – 6 + 3
y = 6x – 3
Step 4: Sketch the curve and the tangent

Example 2: Find the equation of the tangent to f(x) = x3 −3x2 + x−1


Take not that we need to find f(x) to get the coordinates, at the point where x = 3
3 2
f (3)=3 −3(3 )+3−1=27−27+ 3−1=2
So, the point of interest has coordinates (3, 2).
52
College of Engineering Education
2nd Floor, BE Building
Matina Campus, Davao City
Telefax: (082)296-1084
Phone No.: (082)300-5456/300-0647 Local 131

The next thing that we need is the slope of the curve at this point. To find this, we need
to differentiate f (x):
2
f ' (x)=3 x −6 x +1
We can now calculate the slope of the curve at the point where x = 3.
2
f ' (3)=3(3 )−6(3)+1 = 27−18+1=10
So, we have the coordinates of the required point, (3,2), and the slope of the tangent at
that point, 10.
What we want to calculate is the equation of the tangent at this point on the curve. The
tangent must pass through the point and have slope 10. The tangent is a straight line
and so we use the fact that the equation of a straight line that passes through a point
(x1,y1) and has slope m is given by the formula:

Substituting the given values,


y−2
=10
x−3
and rearranging
y−2=10 ( x−3 )
y−2=10 x−30
y=10 x−28 - This is the equation of the tangent to the curve at the point (3,2).

The equation of a normal to a curve


In mathematics, the word ‘normal’ has a very specific meaning. It means ‘perpendicular’
or ‘at right angles’.

Figure 2. The normal is a line at right angles to the tangent.


53
College of Engineering Education
2nd Floor, BE Building
Matina Campus, Davao City
Telefax: (082)296-1084
Phone No.: (082)300-5456/300-0647 Local 131

If we have a curve such as that shown in Figure 2, we can choose a point and draw in
the tangent to the curve at that point. The normal is then at right angles to the curve so
it is also at right angles (perpendicular) to the tangent.
We now find the equation of the normal to a curve. There is one further piece of
information that we need in order to do this. If two lines, having slope m1 and m2
respectively, are at right angles to each other then the product of their slope, m1m2,
must be equal to −1.

Recall that the tangent and the normal are perpendicular. Hence, from the said
key point above, the slope of the normal is just the negative reciprocal of the
slope of the tangent.

Example 1: Determine the equation of the normal to the curve xy=−4 at (−1, 4). Draw a
rough sketch.
Step 1: Find the derivative
Make y the subject of the formula and differentiate with respect to x:
−4
y=
x
−1
y=−4 x
Therefore,
dy −2
=−1 (−4 ) x
dx
' −2
y =4 x
4
y '= 2
x
Step 2: Calculate the slope of the normal at (−1, 4)
First determine the slope of the tangent at the given point:
dy 4
=
dx (−1)2
m=4
Use the slope of the tangent to calculate the gradient of the normal:
m1m2 = -1
(4)(m2) = -1
−1
m2 =
4
54
College of Engineering Education
2nd Floor, BE Building
Matina Campus, Davao City
Telefax: (082)296-1084
Phone No.: (082)300-5456/300-0647 Local 131

Step 3: Find the equation of the normal


Substitute the slope of the normal and the coordinates of the given point
into the slope-point form of the straight-line equation.

y− y 1=m ( x−x 1 )
1
y−4= ( x−(−1) )
4
−1 1
y= x− + 4
4 4
−1 15
y= x+
4 4
or 4 y=−x +15
Therefore, this is the equation of the normal

Step 4: Draw a rough sketch

Example 2: Fnd the equation of the tangent and the equation of the normal to the curve
1
y=x + at the point where x=2.
x
First of all we shall calculate the y coordinate at the point on the curve where x = 2:
1 5
y=2+ =
2 2
dy
Next, we want the gradient of the curve at the point x = 2. We need to find .
dx
55
College of Engineering Education
2nd Floor, BE Building
Matina Campus, Davao City
Telefax: (082)296-1084
Phone No.: (082)300-5456/300-0647 Local 131

Note that we can write y as y=x + x−1 then


dy −2 1
=1−x = 1−¿ 2
dx x
Furthermore, when x = 2
dy 1 3
1−¿ 2 =
dx 2 4
5
This is the gradient of the tangent to the curve at the point (2 , ). We know that the
2
standard equation for a straight line is

With the given values, we have


5
y−
2 3
=
x−2 4

Simplifying,
5 3
y− = ( x−2)
2 4
5
4 ( y − )=3(x−2)
2
4 y−10=3 x −6
4 y=3 x +4

So, the equation of the tangent to the curve at the point where x=2is 4 y=3 x +4
Now we need to find the equation of the normal to the curve.
Let the gradient of the normal be m2. Suppose the slope of the tangent is m1. Recall
that the normal and the tangent are perpendicular and hence m1m2 = −1. We know m1 =
3
. So,
4
3
m2 = -1
4
−4
m2 =
3
So we know the gradient of the normal and we also know the point on the curve through
5
which it passes, (2 , ).
2

56
College of Engineering Education
2nd Floor, BE Building
Matina Campus, Davao City
Telefax: (082)296-1084
Phone No.: (082)300-5456/300-0647 Local 131

5
y−
2 −4
=
x−2 3
Simplifying,

(
3 y−
5
2) =−4 ( x−2 )
15
3 y− =−4 x+ 8
2
15
3 y +4 x=8+
2
31
3 y +4 x=
2
6 y +8 x=31

This is the equation of the normal to the curve at the given point.

Self-Help: Refer to the sources below to help you further understand the
lesson.

Love, C., & Rainville E., (1962). Differential Calculus Notes for Mathematics 100.

e-sources
https://www.siyavula.com/read/maths/grade-12/differential-calculus/06-differential-
calculus-04
http://www.mathcentre.ac.uk/

Let’s Check
Activity 1: Determine the equations of the tangent to the curve defined by:
3 2
1. f ( x )=x +2 x −7 x +1 at x=2
2
2. f ( x )=3 x −2 x+ 4 at x=0∧3
x
3. f ( x )=xe at x=0

Activity 2: Determine the equations of the normal at each of the points indicated.
2
1. f ( x )=x +3 x +1 at x =0∧4
3
2. f ( x )=2 x −5 x+ 4 at x=−1∧1

57
College of Engineering Education
2nd Floor, BE Building
Matina Campus, Davao City
Telefax: (082)296-1084
Phone No.: (082)300-5456/300-0647 Local 131

π
3. f ( x )=tanx at x=
4

Activity 3: Determine the equations of both tangent and normal to the curve:
2
1. y=3 x −x+ 1 at x=1

x−1
2. y= at x=0
x+ 1

Let’s Analyze

Activity 1: For odd-numbered questions, determine the equation of the normal. For
even-numbered questions, determine the equation of the tangent.
2
1. y=3 x −2 x+1 at (1 , 2)
3 2
2. y=x −3 x −2 at (1 ,−4 )
2
3. y=2+ 4 x−x at x=−1
2
4. x −6 x +2 y−8=0 at x=3
2 2
5. x + y −6 x+2 y=0 at ( 0 , 0)
3
6. y=(2 x−1) at x=1

Activity 2: Solve for each of the following:

1. Determine the points where the slope of the tangent to the curve:

a. f ( x )=1−3 x2 is equal ¿5

1 2
b. g ( x )= x + 2 x +1 isequal ¿ 0
3

2. Determine the point(s) on the curve f(x)=(2x−1)2 where the tangent is:

a. parallel to the line y = 4x−2

b. perpendicular to the line 2y + x – 4 = 0.

58
College of Engineering Education
2nd Floor, BE Building
Matina Campus, Davao City
Telefax: (082)296-1084
Phone No.: (082)300-5456/300-0647 Local 131

3. Find the equation of the tangent line at the point (1, 2) with the curve 3 y 2−2 x 5=10.

In a Nutshell
These are some points to remember in solving the equations to tangents and
normal.

 Follow the steps in the module to avoid mistakes in getting the equation.
 You need to familiarize the rules of differentiation for algebraic function to save a
great time in solving for the derivatives. With that, we can get the slope to get the
equation of either the tangent/normal.
 Remember that normal slope is the negative reciprocal of the tangent slope.

Q&A List

If you have any questions regarding tangents and normals, kindly write down on
the table provided.

QUESTIONS ANSWERS
1.

2.

3.

4.

5.

59
College of Engineering Education
2nd Floor, BE Building
Matina Campus, Davao City
Telefax: (082)296-1084
Phone No.: (082)300-5456/300-0647 Local 131

Big Picture in Focus:


ULO-3b. Determine increasing and decreasing functions and its local maximum
and minimum

Metalanguage
This section will serve as your word bank where the most essential terms
relevant to differential calculus and ULO-b will be operationally defined to establish a
common frame of reference. You will encounter these terms as we delve deeper to the
study of Differential Calculus. Please refer to these definitions in case you will find it
difficult to understand mathematical concepts in relation with calculus. Also, you need
the skill in deriving to continue to the next pages.

 A maximum is a high point and a minimum is a low point:

 A function is "increasing" when the y-value increases as the x-value increases,


like this:

60
College of Engineering Education
2nd Floor, BE Building
Matina Campus, Davao City
Telefax: (082)296-1084
Phone No.: (082)300-5456/300-0647 Local 131

 In decreasing functions, the y-value decreases as x-value increases.

 First Derivative Test. A method for determining whether a critical point is a


minimum, maximum, or neither.

Essential Knowledge
In ULO-b, you are expected to be expert in determining increasing and
decreasing functions using the first derivative test.

INCREASING/DECREASING FUNCTIONS
The derivative of a function may be used to determine whether the function is
increasing or decreasing on any intervals in its domain. If f′(x) > 0 at each point in an
interval I, then the function is said to be increasing on I. f′(x) < 0 at each point in an
interval I, then the function is said to be decreasing on I. Because the derivative is zero
or does not exist only at critical points of the function, it must be positive or negative at
all other points where the function exists.
In determining intervals where a function is increasing or decreasing, you first
find domain values where all critical points will occur; then, test all intervals in the
domain of the function to the left and to the right of these values to determine if the
derivative is positive or negative. If f′(x) > 0, then f is increasing on the interval, and if f′
(x) < 0, then f is decreasing on the interval. This and other information may be used to
show a reasonably accurate sketch of the graph of the function.

Example 1: Find where the function, f ( x )=x 3 +3 x 2−9 x +7 is decreasing or increasing

Step 1: Get the first derivative


2
f ' ( x )=3 x +6 x−9

Step 2: Now we want to find the intervals where f ' is positive or negative. This is done
using critical points, which are the points where f ' is either 0 or undefined. f ' is a
polynomial, so it's always defined. To find its zeros, we can factor it:
f ' ( x )=3(x+ 3)(x−1)

Step 3: Find the critical points.

61
College of Engineering Education
2nd Floor, BE Building
Matina Campus, Davao City
Telefax: (082)296-1084
Phone No.: (082)300-5456/300-0647 Local 131

To find these critical points you must first take the derivative of the function.
Second, set that derivative equal to 0 and solve for x.
So, 0=3(x +3)(x−1)
Our crucial points are x = -3 and x = 1. These points divide the number line into
three intervals.

Step 4: Let's evaluate f’ at each interval to see if it's positive or negative on that interval.

(−∞,−3)

(−3 , 1)

(1 ,+∞)

Step 5: Now to find its local maximum and minimum


f’(-4) = +15, f’(0) = -9, f’(2) = +15

+ - +

max

min

To get the y-values of the max and min points, substitute to the original equation
f ( x )=x 3 +3 x 2−9 x +7
3 2
f (−3 )=(−3) +3 (−3) −9(−3)+7 = 34
3 2
f ( 1 ) =(1) + 3(1) −9(1)+7 = 2

Therefore, the maximum point is at (-3, 34) and the minimum point is at (1, 2).
62
College of Engineering Education
2nd Floor, BE Building
Matina Campus, Davao City
Telefax: (082)296-1084
Phone No.: (082)300-5456/300-0647 Local 131

Try …..

For f(x) = x 4 − 8 x 2 determine all intervals where f is increasing or decreasing.

The domain of f(x) are all real numbers, and its critical points occur at x = −2, 0, and 2.
Testing all intervals to the left and right of these values for f′(x) = 4 x 3 − 16 x, you find
that.

Hence, f is increasing on (−2,0) and (2, +∞) and decreasing on (−∞, −2) and (0,2).

Minimum points are at (-2, -16) and (2, -16) while the maximum point is at (0, 0)

Did you also get the same answer? If you did, you are a master!

Self-Help: Refer to the sources below to help you further understand the
lesson.

Love, C., & Rainville E., (1962). Differential Calculus Notes for Mathematics 100.
https://www.cliffsnotes.com/study-guides/calculus/calculus/applications-of-the
derivative/increasing-decreasing-functions
https://www.khanacademy.org/math/ap-calculus-ab/ab-diff-analytical-applications-
new/ab-5-3/a/increasing-and-decreasing-intervals-review

Let’s Check
Activity 1: Find the critical points of the following.

1. f(x)=8x3+81x2−42x−8

2. R(t)=1+80t3+5t4−2t5

3. g(w)=2w3−7w2−3w−2

Activity 2: Let us try to find where a function is increasing or decreasing given the
following.

1. f(x) = x3−4x, for x in the interval [−1,2]


63
College of Engineering Education
2nd Floor, BE Building
Matina Campus, Davao City
Telefax: (082)296-1084
Phone No.: (082)300-5456/300-0647 Local 131

2. h(x)=−x3 + 3x2 + 9

3. y = 2x – 5 on interval ( - ∞, ∞)

Activity 3: Determine the maximum and minimum points of the given functions below.
1. g(x)=x6−2x5+8x4

2. h(z)=4z3−3z2+9z+12

3. f(x) = 3x4 – 16x3 + 24x2

Let’s Analyze

1. Find the range of values of x for which y = x3 + 5x2 - 8x + 1 is increasing.

2. For what values of x is y = 150x - 2x3 an increasing function?

3. Find the intervals in which f (x) = 2x³ + x² - 20 x is increasing or decreasing.

4. Find the intervals in which f (x) = x³ - 3 x + 1 is increasing or decreasing.

5. How will you know if a function increases and decreases and how will you locate the
maximum and minimum value? Explain.

In a Nutshell
These are some points to remember in determining where the function
increases/decreases.

 You should know the first derivative of a function then equate it to 0.


 You must be able to determine the critical points by factoring out the function.
 Set the values of the intervals to determine where the function increases or
decreases.
 To get the maximum, check the graph where it reaches its highest peak and for
the minimum, check where the graph reaches its lowest peak.

Q&A List

If you have any questions regarding this topic, kindly write down on the table
provided.

64
College of Engineering Education
2nd Floor, BE Building
Matina Campus, Davao City
Telefax: (082)296-1084
Phone No.: (082)300-5456/300-0647 Local 131

QUESTIONS ANSWERS
1.

2.

3.

4.

5.

Big Picture
Week 8-9: Unit Learning Outcomes 4 (ULO 4): At the end of the unit, you are
expected to:

a. Solve for concavity: points of inflection using second derivative test.


b. Solve for the Application of Derivatives such as maxima and minima as well
as time rates.

Big Picture in Focus:


ULO-4a. Solve for concavity: points of inflection using second derivative test.

Metalanguage
This section will serve as your word bank where the most essential terms
relevant to differential calculus and ULO-a will be operationally defined to establish a
common frame of reference. You will encounter these terms as we delve deeper to the
study of Differential Calculus. Please refer to these definitions in case you will find it
difficult to understand mathematical concepts in relation with calculus. Also, you need
the skill in deriving to continue to the next pages.

 A maximum is a high point, and a minimum is a low point:

65
College of Engineering Education
2nd Floor, BE Building
Matina Campus, Davao City
Telefax: (082)296-1084
Phone No.: (082)300-5456/300-0647 Local 131

 Inflection points are points where the function changes concavity, i.e. from
being "concave up" to being "concave down" or vice versa.

Essential Knowledge
In ULO-a, you are expected to determine the concavity and the points of
inflection of a function.

Inflection points, concavity upward and downward

A point of inflection of the graph of a function f is a point where the second derivative f′
′ is 0. It’s also a point where the sense of concavity changes.

A piece of the graph of f is concave upward if the curve ‘bends’ upward. For example,
the popular parabola y=x2 is concave upward in its entirety.

A piece of the graph of f is concave downward if the curve ‘bends’ downward. For
example, a ‘flipped’ version y=−x2 of the popular parabola is concave downward in its
entirety.

The relation of points of inflection to intervals where the curve is concave up or down is
exactly the same as the relation of critical points to intervals where the function is
increasing or decreasing. That is, the points of inflection mark the boundaries of the two
different sort of behavior. Further, only one sample value of f′′ need be taken between
each pair of consecutive inflection points in order to see whether the curve bends up or
down along that interval.

In determining intervals where a function is concave upward or concave downward, you


first find domain values where f″(x) = 0 or f″(x) does not exist. Then test all intervals
around these values in the second derivative of the function. If f″(x) changes sign, then (
x, f(x)) is a point of inflection of the function. As with the First Derivative Test for Local
Extrema, there is no guarantee that the second derivative will change signs, and

66
College of Engineering Education
2nd Floor, BE Building
Matina Campus, Davao City
Telefax: (082)296-1084
Phone No.: (082)300-5456/300-0647 Local 131

therefore, it is essential to test each interval around the values for which f″(x) = 0 or
does not exist.

Geometrically, a function is concave upward on an interval if its graph behaves like a


portion of a parabola that opens upward. Likewise, a function that is concave downward
on an interval looks like a portion of a parabola that opens downward. If the graph of a
function is linear on some interval in its domain, its second derivative will be zero, and it
is said to have no concavity on that interval.

Example 1: Determine the concavity of f(x) = x 3 − 6 x 2 −12 x + 2 and identify any


points of inflection of f(x).

Because f(x) is a polynomial function, its domain is all real numbers.

Testing the intervals to the left and right of x = 2 for f″(x) = 6 x −12, you find that

If x = 1  substitute to the 2nd derivative


f(1) = 6(1) – 12 = -6  the negative sign shows that from -∞ to 2 the graph shows a
concavity downward

If x = 3  substitute to the 2nd derivative


f(3) = 6(3)-12 = +6  the positive sign shows that from 2 to + ∞ the graph shows a
concavity upward.

To get the y-coordinate of the inflection point, substitute 2 to the original equation.
f(2) = 23 – 6(22) – 12(2) + 2 = -38

Therefore,
(-∞ to 2)  concave downward
(2 to + ∞)  concave upward
(2, -38)  inflection point

67
College of Engineering Education
2nd Floor, BE Building
Matina Campus, Davao City
Telefax: (082)296-1084
Phone No.: (082)300-5456/300-0647 Local 131

Example 2: Find the values of x for which the curve of y=x 4-4x3 has points of
inflection.

Solution: y’ = 4x3 – 12x2


y’’ = 12x2 – 24x = 12x(x-2)

Set y’’=0, 0 = 12x (x-2);


12x = 0; x-2=0
We get, x = 0 and x = 2

Testing the intervals to the left and right of x = 0 and x = 2 for y’’ = 12x(x-2), you find that

If x = -1  substitute to 2nd derivative


f(-1) = 12(-1) 2 – 24(-1) = +36, the positive sign shows that from -∞ to 0, the graph is
concave upward

If x = 1  substitute to 2nd derivative


f(2) = 12(1) 2 – 24(1) = -12, the negative sign shows that from 0 to 2, the graph is
concave downward

If x = 3  substitute to 2nd derivative


f(3) = 12(3) 2 – 24(3) = 36, the positive sign shows that from 2 to + ∞, the graph is
concave upward

To get the y-coordinate of the inflection points, substitute 0 and 2 to the original
equation.
f(0) = 0
f(2) = 24 – 4(23) = -16

Therefore,
(-∞ to 0)  concave upward
(0 to 2)  concave downward
(2 to + ∞)  concave upward
(0,0) and (2, -16)  inflection points

Self-Help: Refer to the sources below to help you further understand the
lesson.

68
College of Engineering Education
2nd Floor, BE Building
Matina Campus, Davao City
Telefax: (082)296-1084
Phone No.: (082)300-5456/300-0647 Local 131

Feliciano F., and Uy, F., (1983). Differential Calculus and Integral Calculus.
https://www.cliffsnotes.com/study-guides/calculus/calculus/applications-of-the-
derivative/concavity-and-points-of-inflection

Let’s Check
Activity 1: Find the value or values of x for which the curve of the following functions has
a point of inflection.

1. y = (x – 1)4 (x – 6)

2. y = 2x3 – 3x2 - 36x + 25

3. y = 3x4 – 4x3 + 1

4. y = x4 – 4x3 + 4x2

5. y = 3x5 – 15x4 + 20x3 + 3

Activity 2: Find the inflection points of each of the given curve.

1. y = x3 – 3x2 + 4

2. 4y = 3x4 – 16x3 + 24x2

3. 3y = x3 + 3x2 – 9x + 3

6x
4. y =
x+3

5. y = 5x – x5

69
College of Engineering Education
2nd Floor, BE Building
Matina Campus, Davao City
Telefax: (082)296-1084
Phone No.: (082)300-5456/300-0647 Local 131

Let’s Analyze

1. For what values of a and b the point (−1,2) is an inflection point of the graph of the
function y(x) = ax3 + bx2?

In a Nutshell
These are some points to remember in determining your points of inflection in
your function and where your points concave upwards/downwards.

 You should be able to know the 2nd derivative of that function and then equate it
to 0.
 After equating it to 0, you will now get your x-coordinates’ points of inflection.
Then, test the intervals to the left and right.
 Set the values of the intervals to determine the concavity.
 Lastly to get the y-coordinates of your points of inflection, substitute the x-values
to the original equation.

Q&A List

If you have any questions regarding this topic, kindly write down on the table
provided.

QUESTIONS ANSWERS
1.

2.

3.

70
College of Engineering Education
2nd Floor, BE Building
Matina Campus, Davao City
Telefax: (082)296-1084
Phone No.: (082)300-5456/300-0647 Local 131

4.

5.

Big Picture in Focus:


ULO-4b. Solve for the Application of Derivatives such as maxima and minima as well as
time rates

Metalanguage
This section will serve as your word bank where the most essential terms
relevant to differential calculus and ULO-b will be operationally defined to establish a
common frame of reference. You will encounter these terms as we delve deeper to the
study of Differential Calculus. Please refer to these definitions in case you will find it
difficult to understand mathematical concepts in relation with calculus. Also, you need
the skill in deriving and your comprehension to be able to answer the following
problems in the applications of maxima and minima and time rates.

 The terms maxima and minima refer to extreme values of a function, that is, the
maximum and minimum values that the function attains.

 Maximum means upper bound or largest possible quantity. The absolute


maximum of a function is the largest number contained in the range of the
function. For example, the function f(x) = -16x2 + 32x + 6 has a maximum value
of 22 occurring at x = 1. Every value of x produces a value of the function that is
less than or equal to 22, hence, 22 is an absolute maximum. In terms of its
graph, the absolute maximum of a function is the value of the function that
corresponds to the highest point on the graph.

71
College of Engineering Education
2nd Floor, BE Building
Matina Campus, Davao City
Telefax: (082)296-1084
Phone No.: (082)300-5456/300-0647 Local 131

 Conversely, minimum means lower bound or least possible quantity. The


absolute minimum of a function is the smallest number in its range and
corresponds to the value of the function at the lowest point of its graph. If f(a) is
less than or equal to f(x), for all x in the domain of the function, then f(a) is an
absolute minimum. As an example, f(x) = 32x2 - 32x - 6 has an absolute minimum
of -22, because every value of x produces a value greater than or equal to -22.

 In differential calculus, related rates problems involve finding a rate at which a


quantity changes by relating that quantity to other quantities whose rates of
change are known(Wikipedia).

Essential Knowledge
In ULO-b, you are expected to solve the applications of derivatives. Examples
are the applications on maxima and minima and time rates.

MAXIMA AND MINIMA

Steps in Solving Maxima and Minima Problems


1. Identify the constant, say cost of fencing.
2. Identify the variable to be maximized or minimized, say area A.
3. Express this variable in terms of the other relevant variable(s), say A = f(x, y).
4. If the function shall consist of more than one variable, expressed in terms of one
variable (if possible and practical) using the conditions in the problem, say A =
f(x).
5. Differentiate and equate to zero, dA/dx = 0. Why? The roots of the resulting
equation are the critical numbers which will give the desired maximum or
minimum value of the function

The process of finding maximum or minimum values is called optimization. We are


trying to do things like maximize the profit in a company, or minimize the costs, or find
the least amount of material to make a particular object

These are very important in the world of industry(M.Bourne).

Example 1: The daily profit, P in $, of an oil refinery is given by


P = 8x − 0.02x2,

72
College of Engineering Education
2nd Floor, BE Building
Matina Campus, Davao City
Telefax: (082)296-1084
Phone No.: (082)300-5456/300-0647 Local 131

where x is the number of barrels of oil refined. How many barrels will give maximum
profit and what is the maximum profit?

Solution: In this problem, the equation is already given so, let us proceed to step 5.
dP
The profit is a max (or min) if =0
dx
dP
=8−0.04 x (2)
dx
0=8−0.04 x
x=200 → Is this a maximum?
Remember one way of testing if it’s a maximum is to assume values to the left and to
the right of the value 200.

So, substituting x as 199 to equation (2), we have a (+) value. Substituting x as 201 to
equation 2 yields us a (–) value.

max

We can conclude that the value x = 200 is a maximum

If x = 200, then we have the final answer as P = 8(200) – 0.02(2002) = 800$


So, if the company refines 200 barrels per day, the maximum profit of 800 is reached.

Example 2: A long strip of tin 30 cm wide is to be made into a gutter with rectangular
cross section by turning up equal widths along the edges. Find the depth of the gutter
which yields the greatest carrying capacity.

Step 1:
Solution: Let x - depth of the gutter
y - base of the rectangular cross section
A - area of the rectangular cross section

Step 2 - 4:
To ensure the greatest carrying capacity, we must make the area of the cross section
as great as possible. That is, we maximize A. Thus
A = xy  to be maximized
73
College of Engineering Education
2nd Floor, BE Building
Matina Campus, Davao City
Telefax: (082)296-1084
Phone No.: (082)300-5456/300-0647 Local 131

2x + y = 30 (2)
From equation (2), we get
y = 30 – 2x (3)
Substituting (3) in (1), we obtain
A = x(30 – 2x) = 30x – 2x2
Step 5:
Differentiating with respect to x then equate to 0
dA
=30−4 x
dx
0=30−4 x
x=7.5 cm →The depth of the gutter which yields the greatest carrying capacity.

TIME RATES

 If a quantity x is a function of time t, the time rate of change of x is given by dx/dt.


 When two or more quantities, all functions of t, are related by an equation, the
relation between their rates of change may be obtained by differentiating both
sides of the equation with respect to t.

STEPS IN SOLVING TIME RATES


1. Draw a figure when necessary
2. Define the given values
3. Formulate Equations
4. Differentiate with respect to time
5. Substitute the condition/s to the equation
74
College of Engineering Education
2nd Floor, BE Building
Matina Campus, Davao City
Telefax: (082)296-1084
Phone No.: (082)300-5456/300-0647 Local 131

Example 1: Water is flowing into a vertical cylindrical tank at the rate of 24 ft 3/min. If the
radius of the tank is 4 ft, how fast is the surface rising?

Step 1: Draw a figure when necessary

Step 2: Define the given values


radius = 4 ft
dV 3
=24 ft /min
dt
dh
=?
dt
Step 3: Formulate equations
2 2 2
V =π r h=π ( 4 ft) h=16 ft πh
Step 4: Differentiate with respect to time
dV dh
=16 π
dt dt
dV 3
Since =24 ft /min
dt
3 2 dh
24 ft /min=16 ft π
dt
dh
=0.477 ft /min
dt
Therefore, the surface is rising at 0.477ft/min.

Example 2: Water flows into a vertical cylindrical tank at 12 ft3/min, the surface rises 6
in/min. Find the radius of the tank.

Now, what we are going to find is the radius.

Step 1: Draw a figure when necessary

75
College of Engineering Education
2nd Floor, BE Building
Matina Campus, Davao City
Telefax: (082)296-1084
Phone No.: (082)300-5456/300-0647 Local 131

Step 2: Define the given values


radius = ?
dV 3
=12 ft /min
dt
dh
=6∈¿ min ¿ 0.5 ft /min
dt

Step 3: Formulate equations


2
V =π r h
Step 4: Differentiate with respect to time
dV 2 dh
=π r
dt dt

dV 3
Since =12 ft /min
dt
3 2
12 ft /min=π r ( 0.5 ft / min)
2 2
r =7.639437268 ft
r =2.76395 ft
Therefore, the radius of the tank is approximately 2.76 ft.

Self-Help: Refer to the sources below to help you further understand the
lesson.

Feliciano F., and Uy, F., (1983). Differential Calculus and Integral Calculus.
Love, C., & Rainville E., (1962). Differential Calculus Notes for Mathematics 100.
https://www.intmath.com/applications-differentiation/7-maximum-minimum-
problems.php.

Let’s Check
Activity 1: Applications of Maxima and Minima

1. Find two nonnegative numbers whose sum is 9 and so that the product of one
number and the square of the other number is a maximum.

2. A rectangular box with a square base and no top is to have a volume of 108
cubic inches. Find the dimensions for the box that require the least amount of
material.
76
College of Engineering Education
2nd Floor, BE Building
Matina Campus, Davao City
Telefax: (082)296-1084
Phone No.: (082)300-5456/300-0647 Local 131

3. An open rectangular box with square base is to be made from 48 ft.2 of material.
What dimensions will result in a box with the largest possible volume?

Activity 2: Applications of Time Rates

1. Air is being pumped into a spherical balloon such that its radius increases at a
rate of .50 in/min. Find the rate of change of its volume when the radius is 3
inches.

2. A ladder 10 meters long rests on horizontal ground and leans against a vertical
wall. The foot of the ladder is pulled away from the wall at the rate of 0.3 m/sec.
How fast is the top sliding down the wall when the foot of the ladder is 2 m from
the wall?

3. A baseball diamond is a square 90 ft on a side. A player runs from first base to


second base at 15 ft/sec. At what rate is the player's distance from third base
decreasing when she is half way from first to second base?

Let’s Analyze

1. Find the dimensions of a rectangle with perimeter 1000 meters so that the area of the
rectangle is a maximum.

2. A closed right circular cylindrical tank is to have a capacity of 128 π m3 . Find the
dimensions of the tank that will require the least amount of material in making it.

3. The radius of a right circular cone is increasing at the rate of 6 cm/sec while its
altitude is decreasing at 3 cm/sec. Find the rate of change of its volume when its radius
is 8 cm and its altitude is 20 cm

4. Water is running out of a conical tank 3 m across the top and 4 m deep at the rate of
2 m3 /min. Find the rate at which the level of water drops when it is 1 m from the top.

In a Nutshell
Here are the important steps that you always need to remember:

Steps in Solving Maxima and Minima Problems


1. Identify the constant, say cost of fencing.

2. Identify the variable to be maximized or minimized, say area A.

3. Express this variable in terms of the other relevant variable(s), say A = f(x, y).
77
College of Engineering Education
2nd Floor, BE Building
Matina Campus, Davao City
Telefax: (082)296-1084
Phone No.: (082)300-5456/300-0647 Local 131

4. If the function shall consist of more than one variable, expressed in terms of one
variable (if possible and practical) using the conditions in the problem, say A =
f(x).
5. Differentiate and equate to zero, dA/dx = 0. Why? The roots of the resulting
equation are the critical numbers which will give the desired maximum or
minimum value of the function

Steps in solving Time Rates Problems


1. Draw a figure when necessary

2. Define the given values

3. Formulate Equations

4. Differentiate with respect to time

5. Substitute the condition/s to the equation

Q&A List

If you have any questions regarding the applications of time rates and maxima &
minima, kindly write down on the table provided.

QUESTIONS ANSWERS
1.

2.

3.

4.

5.

78
College of Engineering Education
2nd Floor, BE Building
Matina Campus, Davao City
Telefax: (082)296-1084
Phone No.: (082)300-5456/300-0647 Local 131

Big Picture in Focus:


ULO-5a. Perform differentiation of trigonometric functions for the application of
maxima and minima

Big Picture
Week 9-10: Unit Learning Outcomes 5 (ULO-5): At the end of the unit, you are
expected to:
a. Perform differentiation of trigonometric functions for the application of maxima and
minima;
b. Apply differentiation of inverse trigonometric functions to solve for optimization
problems;

Metalanguage
This section will serve as your word bank where the most essential terms
relevant to ULO5-a will be operationally defined to establish a common frame of
reference. You will encounter these terms as we delve deeper to the study of
Differential Calculus regarding transcendental functions.
1. Transcendental Function. It is a function that cannot be expressed as a finite
combination of the algebraic operations, such as addition, subtraction, multiplication,
division, raising to a power, and extracting a root.

79
College of Engineering Education
2nd Floor, BE Building
Matina Campus, Davao City
Telefax: (082)296-1084
Phone No.: (082)300-5456/300-0647 Local 131

1.1 In general, the term transcendental means non-algebraic which is not expressible
as a solution of a polynomial equation.
2. Trigonometric Functions. These are functions that describe the relation between
the sides and angles of a right triangle.
2.1 This kind of function includes the following six (6) functions: sine, cosine,
tangent, cotangent, secant, and cosecant. For each of these functions, there is an
inverse trigonometric function.

Essential Knowledge
In ULO-5a, you will run across various formulas for the differentiation of
trigonometric functions that you need to familiarize since this will be necessary in
solving for the maxima and minima problems. It is suggested to practice solving
problems involving trigonometric functions so that it would be easier for you to recall the
formulas.

For the entire two weeks, we will be dealing with the transcendental functions
such as the trigonometric, inverse trigonometric, exponential, logarithmic, and
hyperbolic functions. The differentiation of trigonometric functions will be discussed in
this Unit Learning Outcome (ULO).

1. Differentiation of Trigonometric Functions. The formulas shown on a table on the


next page are used for differentiating trigonometric functions. The symbol u denotes
an arbitrary differentiable function of x .

DIFFERENTIATION OF
TRIGONOMETRIC FUNCTIONS
d du
( sin u )=cos u
dx dx
d du
( cos u )=−sin u
dx dx
d du
( tanu )=sec 2 u
dx dx
d du
( cot u ) =−csc 2 u
dx dx
d du
( secu )=sec u tanu
dx dx
d du
( csc u )=−csc u cotu
dx dx

80
College of Engineering Education
2nd Floor, BE Building
Matina Campus, Davao City
Telefax: (082)296-1084
Phone No.: (082)300-5456/300-0647 Local 131

To find the derivative of a certain trigonometric function, you must follow the
formulas shown on the table. Study the examples given below and be guided with the
table:

dy
Example 5a.1: Find if y=sin 4 x .
dx

Solution: Take note that y=sin 4 x is taken from the form y=sinu where u=4 x .
dy d
= ¿ where: u=4 x
dx dx
dy d
=cos 4 x (4 x ) derive 4 x
dx dx
dy
=cos 4 x (4)
dx
dy
=4 cos 4 x
dx

dy
Example 5a.2: Find if y=sin3 4 x .
dx
Solution: The given function has a power; hence we need to use the general power formula
to find its derivative.
d n
'
y =( 3 ) ¿ ¿ general power formula: ( u ) =n(u n−1) du
dx dx
d du
'
y =3 ¿ ( sin u )=cos u
dx dx
'
y =3 ¿ simplify by multiplying

' 2
y =12 sin 4 x cos 4 x

Example 5a.3: Find g '(x ) if g ( x )=3 sec x−10 cot x .


Solution: Refer to the table of the trigonometric function derivatives and differentiate each
term using the formulas there.

( dxd sec x )( dxd x)−( 10) ( dxd cot x)( dxd x)


g' ( x ) = ( 3 )

g ( x ) =( 3 ) ( sec x tan x ) ( x )−( 10 ) (−csc x ) ( x )


' d 2 d
dx dx

81
College of Engineering Education
2nd Floor, BE Building
Matina Campus, Davao City
Telefax: (082)296-1084
Phone No.: (082)300-5456/300-0647 Local 131

g' ( x ) =( 3 ) ( sec x tan x ) ( 1 )−( 10 ) (−csc 2 x ) ( 1 )


g' ( x ) =( 3 ) ( sec x tan x ) ( 1 ) + ( 10 ) ( csc 2 x ) ( 1 )
' 2
g ( x ) =3 sec x tan x +10 csc x
 You can isolate the constants, 3 and 10, first from each term before differentiating the function.
Be careful with the minus sign in front of the second term and make sure it is properly simplified
by combining the minus sign and the negative sign of the second term.

Example 5a.4: Find h '(w) if h ( w )=3 w−4 −w 2 tan w .


Solution: The product rule of differentiation must be used on the second term of the
function.

h ' ( w )=(3)(−4)(w¿¿−4−1) ( dwd w )−¿ ¿


h ' ( w )=(−12)(w¿¿−5) ( 1 )−¿ ¿
−5
h ' ( w )=−12 w −¿
−5 2 2
h ' ( w )=−12 w −(w sec w +2 w tan w)
−5 2 2
h ' ( w )=−12 w −(w sec w +2 w tan w)
−5 2 2
h ' ( w)=−12 w −w sec w−2 w tan w
 The second term of the function requires the product rule to find its derivative. To avoid
confusion while deriving with the minus sign in front of the second term, we need parentheses
to properly group the terms since the second term will produce two terms after differentiating.
After performing differentiation to all terms, make sure you distribute the minus sign to the
terms inside the parentheses to get the simplified final answer.
Example 5a.5: Find y ' if y=5 sin x cos x +4 csc x .
Solution: The product rule of differentiation must be used on the first term of the function
and we will isolate the constants, 5 and 4, before differentiating the first and second
terms.
'
y =( 5 ) ¿
'
y =( 5 ) ¿
'
y =( 5 ) ¿
'
y =( 5 ) ¿
' 2 2
y =−5 sin x+ 5 cos x−4 csc x cot x rearrange the terms

Or
' 2 2
y =5 cos x −5 sin x−4 csc x cot x

82
College of Engineering Education
2nd Floor, BE Building
Matina Campus, Davao City
Telefax: (082)296-1084
Phone No.: (082)300-5456/300-0647 Local 131

 Isolating the constants before performing differentiation will make the process easier provided that the
constant is part of the product of a term like the constants 5 and 4 in the first and the second term,
respectively.

sin t
Example 5a.6: Find P '(t ) if P(t )= .
3−2 cos t
Solution: The quotient rule of differentiation must be used on this problem. Let u=sin t
and v=3−2cos t .
'
P ( t )=¿ ¿
'
P ( t )=¿ ¿
'
P ( t )=¿ ¿
'
P ( t )=¿ ¿
2 2
' 3 cos t−2 cos t−2 sin t
P ( t )= factor out (−2) from
( 3−2 cos t )2
2 2
−2 cos t−2 sin t
'
P ( t )=3 cos t−2¿ ¿ use the identity: sin
2 2
x+ cos x=1
' 3 cos t−2(1)
P ( t )=
( 3−2cos t )2
' 3 cos t−2
P ( t )=
( 3−2cos t )2

 A trigonometric identity is used to simplify the final answer of the problem. There are times that you need
to simplify further a trigonometric function based on various trigonometric identities.

A table of selected trigonometric identity is laid out for you as a reference for
future trigonometric problems you will encounter. These identities are assumed to be
the most common one.

TRIGONOMETRIC IDENTITIES
RECIPROCAL IDENTITIES DOUBLE-ANGLE IDENTITIES
1 cos x
cot x= = sin 2 x=2 sin x cos x
tan x sin x
2 tan x
¿
1 2
1+ tan x
csc x=
sin x
2 2
cos 2 x=cos x−sin x
1
sec x= ¿ 2 cos x−1
2
cos x
2
¿ 1−2 sin x

83
College of Engineering Education
2nd Floor, BE Building
Matina Campus, Davao City
Telefax: (082)296-1084
Phone No.: (082)300-5456/300-0647 Local 131

PYTHAGOREAN IDENTITIES
2 2
sin x+ cos x=1
2 2
sin x=1−cos x
2 2
cos x=1−sin x 1−tan x
2
¿ 2
1+ tan x=sec x
2 2 1+tan x
2 2
1+cot x=csc x 2 tan x
tan2 x= 2
1−tan x
2
sin x
Example 5a.7: Find y ' if y= 2 .
cos x
Solution: This problem requires simplification before differentiation. Please refer to the
table of trigonometric identities to simplify the function.
2
sin x
y= 2
cos x

( )
2
sin x
y=
cos x
2 2
y= ( tan x ) =tan x apply general power formula

y '=2¿
y '=2¿

y '=2¿
' 2
y =2 sec x tan x

Example 5a.8: Find y ' if y=sin2 x +cos 2 x


Solution: This problem can be solved in various ways, but the most straightforward
process is used here.
' d
y= ¿
dx

y ' =cos ( 2 x )
[ d
dx ]
(2 x) +(2)¿ ¿

'
y =cos ( 2 x ) (2)+(2) ¿ ¿

84
College of Engineering Education
2nd Floor, BE Building
Matina Campus, Davao City
Telefax: (082)296-1084
Phone No.: (082)300-5456/300-0647 Local 131

'
y =2 cos 2 x−2 sin x cos x use identity:
sin 2 x=2 sin x cos x
'
y =2 cos 2 x−sin 2 x

Example 5a.9: Find y ' if y=tan(sin x )


Solution: The given equation is not a product of functions; it is what we call
composition of functions.

' d
y= ¿ note: u=sin x
dx

y ' =sec 2 (sin x)


[ d
dx
(sin x )
]
' 2
y =sec ( sin x ) ¿

Example 5a.10: Find y ' if y=cos3 ( tan(3 x))


Solution: Apply series of chain rule to differentiate the function.

y '=(3)¿¿

y '=(3)¿¿

y '=(3)¿¿

y '= ( 3 ) ¿ ¿
2
y '=−9 cos ¿ ¿ ¿

After familiarizing the process of trigonometric function derivation, we need to


study how this method works when used in the application of maxima and minima.
There are different word problems to be dealt with when talking about maxima and
minima, so it is advised to practice solving these kinds of applications.

Example 5a.11: A trough for holding water is formed by taking a piece of sheet metal
60 cm wide and folding the 20 cm on either end up as shown below. Determine the
angle θ that will maximize the amount of water that the trough can hold.

85
College of Engineering Education
2nd Floor, BE Building
Matina Campus, Davao City
Telefax: (082)296-1084
Phone No.: (082)300-5456/300-0647 Local 131

Solution: To solve this problem, we need to maximize the volume of the trough. But if you think about
calculating the volume of a trough, it would be the cross-sectional area multiplied by the length. So,
whatever the given length is, we can maximize the volume of the trough my maximizing its cross-sectional
area. Let us redo the sketch of the trough to get the formula of the cross-sectional area.

With this more specific sketch, we can say that the cross-sectional area of the trough has:
 a rectangle in the middle with height h and width 20
 two (2) identical right triangle with height h , base b , and hypotenuse 20

Note: In basic geometry, the angle between the base b and hypotenuse 20 is equal to the angle θ of the
original sketch.
Let us now start formulating the values of b and h in terms of θ :
b h
cos θ= sin θ=
20 20
b=20 cos θ h=20 sin θ
Now, based on the new sketch, the cross-sectional area of the trough in terms of θ is:
A=area of rectangle+area of triangle+area of triangle
1 1 1
A=lw+ bh+ bh=lw +2 bh
2 2 2 ( )
A=20(20 sin θ)+2 ¿
A=400 sin θ+ 400 cos θ sin θ
A=400 ¿

Let us now maximize the cross-sectional area by performing differentiation:

A ' (θ)=(400) [ cos θ +sin θ (−sin θ ) +cos θ (cos θ) ]


A ' (θ)=(400) [ cos θ−s ¿ θ+cos θ ] 2 2
2 2
use trig identity: s ¿ θ=¿ 1−c os θ ¿
A ' (θ)=(400)¿
A ' (θ)=(400)¿

86
College of Engineering Education
2nd Floor, BE Building
Matina Campus, Davao City
Telefax: (082)296-1084
Phone No.: (082)300-5456/300-0647 Local 131

2
A ' (θ)=(400)¿ factor out: 2 c os θ+ ¿ cos θ−1¿
A ' (θ)=(400) ( 2 cos θ−1 ) (cos θ+ 1)
Let A' ( θ )=0

0=(400) ( 2 cos θ−1 ) (cos θ+ 1)

( 4001 )0=[(400) ( 2cos θ−1) (cos θ+1)]( 4001 )


0=( 2 cos θ−1 ) (cos θ+1)
Find values of θ :
2 cos θ−1=0 cos θ+ 1=0
2 cos θ=1 cos θ=−1
1 −1
cos θ= θ=cos (−1 )
2
−1 1
θ=cos θ2=π
2
π
θ1=
3

We have come up with two (2) values of θ , however, it won’t make sense if we consider θ2=π
π
(180 °∈degree) since θ must be in the interval of 0 ≤ θ ≤ or in terms of degrees, 0 ≤ θ ≤90 °
2
π
.Therefore, the maximum angle θ that will maximize the amount of water that the trough can hold is .
3

Self-Help: Refer to the sources below to help you further understand the
lesson.

Feldman, J., & Rechnitzer, A. (2015). Differential Calculus Notes for Mathematics 100.

Lax, P. D., & Terell, M. S. (2014). Calculus with Applications. New York City: Springer.

Terano, H. J. (2015). Calculus 1: A simplified Text in Differential Calculus.

Let’s Check
87
College of Engineering Education
2nd Floor, BE Building
Matina Campus, Davao City
Telefax: (082)296-1084
Phone No.: (082)300-5456/300-0647 Local 131

Activity 1. Find the derivative of the following trigonometric functions and simplify if possible.
Box your final answer.

1. y=3 sin x−4 cos x

2. y=x 3 tan x

cos x
3. f (x)=
1+sin x

4. y=csc x cot x

5. f (x)=cos 2 x+ sin2 x

sin 3 x
6. y=
4+5 cos (2 x )

7. y=x sec 2( πx )

x csc x
8. g ( x )=
3−csc x

Let’s Analyze
Activity 1. Solve for the derivative of the following and simplify if possible.

1. f ( x )=2cos x −6 sec x +3
2. f ( w )=tan w sec w

3. h ( t )=t 3−t 2 sin t

4. y=6+4 √ x csc x

1
5. R(t )=
2sin t−4 cos t

Activity 2. Solve the following problems and write the givens, what to maximize, and
the solution. Box your final answer.

1. Find the height of a right circular cylinder of maximum volume that can be
inscribed in a sphere of radius 15 cm.

2. Find the length of the shortest ladder which will reach from the ground level to a
high vertical wall if it must clear an 8-ft vertical fence which is 27 ft from the wall.
88
College of Engineering Education
2nd Floor, BE Building
Matina Campus, Davao City
Telefax: (082)296-1084
Phone No.: (082)300-5456/300-0647 Local 131

3. A man in a motorboat at A receives a message at noon, calling him to B. A bus


making 40 miles per hour leaves C, bound for B, at 1:00 PM. If AC = 30 miles,
what must be the speed of the boat, to enable the man to catch the bus?

4. A gutter having a triangular cross-section is to be made by bending a strip of tin


in the middle. Find the angle between the sides when the carrying capacity is to a
maximum.

5. A trapezoidal gutter is to be made, from a strip of metal 22 inches wide by


bending up the edges. If the base is 14 inches wide, what width across the top
gives the greatest carrying capacity.

In a Nutshell
These are some points to remember if you are going to derive trigonometric
functions:

 The derivative of the six (6) trigonometric functions can easily be determined by
using their predetermined formulas.
 Trigonometric identities may be used to simplify the trigonometric function.
 The rules for differentiating algebraic functions are used in performing derivation
to trigonometric functions.

Q&A List

If you have any questions regarding continuity, kindly write down on the table
provided.

QUESTIONS ANSWERS
1.

2.

3.

4.

5.

89
College of Engineering Education
2nd Floor, BE Building
Matina Campus, Davao City
Telefax: (082)296-1084
Phone No.: (082)300-5456/300-0647 Local 131

Big Picture in Focus:


ULO-5b. Apply differentiation of inverse trigonometric functions to solve for
optimization problems

Metalanguage
In this section, you will find terms that are related to the differentiation of inverse
trigonometric functions. These terms will be your basis as to what they are used for or
how they are being used in this concept. You may go back here anytime if you run
across a certain term you find ambiguous.
1. Inverse Trigonometric Functions. These functions are used to determine the
angle measure when at least two (2) sides of a right triangle are known.
a. These functions have practical uses in navigation, physics, engineering, and
other sciences.
b. These functions give the value of the angle in degrees ( ° ) or in radians from
its Trigonometric Function Value.
Essential Knowledge
In ULO3b, you are expected to derive inverse trigonometric functions correctly by
using certain formulas that will not require you to use the long-method differentiation.
Equipped with the knowledge of the rules for differentiating algebraic functions using its
specified rules, you can achieve the intended learning outcome. These are the things
you need to know in addition to the first transcendental function we discussed on the
previous ULO.
1. Inverse Trigonometric Functions. The six (6) inverse trigonometric functions
namely sin−1 x ,cos−1 x , tan−1 x , csc−1 x , sec−1 x ,∧cot−1 x are used to find the unknown
measure of an angle of a right triangle when two (2) side lengths are known. In this
text, we shall denote our inverse trig functions with “Arc”.

Similar to the process of differentiating Trigonometric Functions, Inverse Trig


Functions can be derived using formulas which will be shown in the table.
DIFFERENTIATION OF INVERSE TRIGONOMETRIC FUNCTIONS
d 1 du d −1 du
( arcsin u )= ( arccot u )=
dx √1−u dx
2
dx 1+u dx
2

d −1 du d 1 du
( arccos u )= ( Arcsec u )=
dx √ 1−u2 dx dx u √ u −1 dx
2

d 1 du d −1 du
( arctan u )= ( Arccsc u )=
dx 1+ u dx dx u √ u 2−1 dx
2

90
College of Engineering Education
2nd Floor, BE Building
Matina Campus, Davao City
Telefax: (082)296-1084
Phone No.: (082)300-5456/300-0647 Local 131

Examples on how to use the formulas for differentiating the inverse trigonometric
functions will be presented on the succeeding pages.

dy
Example 5b.1: Find if y=arcsin 3 x
dx
Solution: The function y=arcsin 3 x takes the form y=arcsin u where u=3 x .
dy 1
=
dx √1−¿ ¿¿
dy 1
=
dx √ 1−¿ ¿¿
dy 3
=
dx √1−9 x 2

dy x
Example 5b.2: Find if y=arctan
dx 4
Solution:

dy
dx
=
1+
1

()
x [ ( )]
2
d x
dx 4
4
dy
dx
=
1

1+ 2
x
2 [ ( )] 1 d
4 dx
x

4
dy
dx
=
1+
1
x2 [ ( )]
1
4
1 =
( )
1+
1
x2 ( 14 )
16 16

[( ) ] (
dy 1 1
= =
dx
)
2
x 4 x2
1+ 4 4+
16 16
dy 1 1
= =
dx [ 4 (16)+ 4 x ] 4 (16+ x 2)
2

dy 4
=
dx 16 + x 2

Example 5b.3: Find y ' if y=arcsin( x−1)


Solution:

91
College of Engineering Education
2nd Floor, BE Building
Matina Campus, Davao City
Telefax: (082)296-1084
Phone No.: (082)300-5456/300-0647 Local 131

dy
=
1 d
dx √1−(x−1) dx
2
(x −1)
[ ]
dy 1
= (1)
dx √ 1−(x −2 x+1)
2

dy 1 1
= =
dx √ 1−x +2 x−1 √−x 2 +2 x
2

dy 1
=
dx √ 2 x−x 2

Example 5b.4: Find y ' if y=arctan ( x−1


x +1
)
Solution:

y '=
1+
1
x +1
( )
[2( )]
d x +1
dx x−1
x−1

y '=
1+
1
( x+ 1 )[2
( x−1 )( 1 )−(x +1)(1)
(x−1)2 ]
( x−1 )2
( x−1 ) ( 1 )−(x+ 1)(1)
y '=

( 1+
( x+ 1 )2
( x−1 ) 2)(x−1)
2

x−1−x−1
y '= 2 2
(x−1) + ( x +1 )
−2
y '= 2 2
x −2 x+1+ x +2 x+ 1
−2
y '= 2
2 x +2

−1
y '= 2
x +1

Example 5b.5: Find y ' if y=


2
√3
arccot
(
2 x+1
√3 )
Solution:
92
College of Engineering Education
2nd Floor, BE Building
Matina Campus, Davao City
Telefax: (082)296-1084
Phone No.: (082)300-5456/300-0647 Local 131

[ ]) [ ( )]
' 2 −1 d 2 x+ 1
y=
√3 dx √ 3
(
2
2 x+1
1+
√3

[ [] ( ) ]
' −2 1 1 d
y= ( 2 x+1 )
√3 ( 2 x +1 )2 √ 3 dx
1+ 2
(√ 3)

( )( )
−2 1 2
y'=
√3 1+
2
4 x + 4 x+ 1 √3
3

( )
−4 1
y'=
3 3+ 4 x + 4 x +1
2

3
'
y=
−4
( 3
3 4 x 2+ 4 x +4 )
' −1
y= 2
x + x +1

At this point, we will solve a problem using the differentiation of inverse


trigonometric function.

Example 5b.6: A ladder 25 ft long leans against a vertical wall. If the lower end is
pulled away at the rate of 6 ft/sec, how fast is the angle between the ladder and the
floor changing when the lower end is 7ft from the wall?

Solution: A sketch must be done first to deeply understand the problem. A label based on the givens from
the problem is also necessary.
25
ft
Note: Let θ be the angle
between the ladder and the

dx
=6 ft /sec θ
dt
x
93
College of Engineering Education
2nd Floor, BE Building
Matina Campus, Davao City
Telefax: (082)296-1084
Phone No.: (082)300-5456/300-0647 Local 131

7 ft

dθ dx
We need to find when =6 ft /sec and x=7 ft . To do this, we shall use the angle θ ,
dt dt
x
hypotenuse, and x of the right triangle made by the ladder, floor, and wall. Use cos θ= . Find θ by
25
isolating it on the left side of the equal sign (=). Thus, we have:

x
θ=arccos
25


Differentiate the equation with respect to t to compute for :
dt


=
−1
[ ( )]
d x

√ 25 )
dt dt 25
(
2
x
1−


=
−1
( )( )
1 dx


dt x 25 dt 2
1−
625

dx
Now, substitute the values of x=7 and =6.
dt

=
−1
( 1
) ( 6)

dt (7) 25
2
1−
625
dθ −1
= rad / sec
dt 4

The negative sign tells us that the angle θ is decreasing.

Self-Help: Refer to the sources below to help you further understand the
lesson.

Feldman, J., & Rechnitzer, A. (2015). Differential Calculus Notes for Mathematics 100.
Lax, P. D., & Terell, M. S. (2014). Calculus with Applications. New York City: Springer.
Terano, H. J. (2015). Calculus 1: A simplified Text in Differential Calculus.

94
College of Engineering Education
2nd Floor, BE Building
Matina Campus, Davao City
Telefax: (082)296-1084
Phone No.: (082)300-5456/300-0647 Local 131

Let’s Check

Activity 1. Find the derivative of the following inverse trigonometric functions and simply it
whenever possible.

x
1. y=arccos
1−x

4
2. y=arctan
x

3. y=arccot ¿

4. y= Arcsec √ 4 x+1

x
5. y= Arccsc
2

Let’s Analyze

Activity 1. Differentiate the following functions and simplify if possible.

1 4 sin x
1. y= arctan
4 3+ 5cos x

2. y=arccos − √
x 4−x 2
2 x
1
3. y=x √ 1−4 x + arcsin 2 x
2
2
2+ x
4. y=arccot x +arctan
1−2 x

5. y= ( x −1 ) √ 2 x−x 2−arccos(x−1)

Activity 2. Solve the following problems using inverse trigonometric functions.

1. The lower edge of a picture is 4 ft., the upper edge is 9 ft above the eye of an observer.
At what horizontal distance should he stand if the angle subtended by the picture is a
maximum?

95
College of Engineering Education
2nd Floor, BE Building
Matina Campus, Davao City
Telefax: (082)296-1084
Phone No.: (082)300-5456/300-0647 Local 131

2. An isosceles triangle has legs 10 cm. The base decreases at the rate of 4 cm/sec. Find
the rate of change of the angle at the apex when the base is 16 cm.

3. A ladder 14 ft long is leaning against a fence 8 ft high, with the upper end projecting over
the fence. If the lower end slides away from the fence at the rate of 2 ft/sec, find the rate
at which the angle between the ladder and the ground is changing when the upper end
is just at the top of the fence.
In a Nutshell
You need to take note of these points when dealing with the differentiation of
inverse trigonometric functions.

 A set of formulas for differentiating inverse trigonometric functions exist to avoid


using the long method.
 Inverse trigonometric functions are necessary to determine the angle of a triangle
with known 2 sides.
 Inverse trigonometric function is the opposite of the trigonometric functions.
These functions are namely sin−1 x ,cos−1 x , tan−1 x , csc−1 x , sec−1 x , ¿ cot−1 x.

Q&A List

If you have any questions regarding continuity, kindly write down on the table
provided.

QUESTIONS ANSWERS
1.

2.

3.

4.

5.

Big Picture
Week 11-12: Unit Learning Outcomes 6 (ULO-6): At the end of the unit, you are
expected to:
a. Solve for the derivative of exponential and logarithmic functions; and

96
College of Engineering Education
2nd Floor, BE Building
Matina Campus, Davao City
Telefax: (082)296-1084
Phone No.: (082)300-5456/300-0647 Local 131

b. Familiarize the properties of the hyperbolic functions and find its derivative
Metalanguage
The terms that will be useful in this unit learning outcome are defined in this
section. These terms are all related as basis of logarithmic and exponential functions.
You may go back here anytime if you run across a certain term you find ambiguous.

Big Picture in Focus:


ULO-3c. Solve for the derivative of exponential and logarithmic functions

1. Logarithmic Function. This function is the inverse of exponential function, where


all exponential functions can be expressed as logarithmic functions.
1.1 This function is necessary in permitting us to work with very large numbers while
manipulating numbers of a much more manageable size.

2. Exponential Function. This function is the opposite of the logarithmic functions. All
logarithmic functions can be expressed as exponential functions.
2.1 This function has an equation where the variable appears as the exponent.

Essential Knowledge
In ULO6a, you are expected to recall your knowledge about the properties of the
logarithmic and exponential functions. These properties will be of great help when
dealing with the differentiation of these functions. The logarithmic and exponential
functions are the 3rd and 4th transcendental functions to be differentiated.

1. The Logarithmic and Exponential Functions. Logarithms and exponents work


well together because they “undo” each other, as long as the base is the same. They
can be called as “Inverse Functions.”

A logarithmic function is defined by:


y=log b x
On the other hand, an exponential function is defined by the equation:
y
x=b
Where in both forms, x >0 , b>0 and b ≠ 1. Putting them side by side, we can
conclude that they are equivalent to one another.

y=log b x x=b
y

the base remains the base

97
College of Engineering Education
2nd Floor, BE Building
Matina Campus, Davao City
Telefax: (082)296-1084
Phone No.: (082)300-5456/300-0647 Local 131

The laws of exponents, radicals, and logarithms have been part of your study in
Algebra and Trigonometry. Some of these laws are necessary to aid in our present
topic and it is listed down here for easy reference.

Laws of Exponents:

E1. a m an=am+ n
m
a m−n
E2. n
=a where m>n , a ≠ 0
a
n
E3. ( a m ) =amn

E4. ( ab )n=a n b n

()
n n
a a
E5. = n where b ≠ 0
b b

Laws of Radicals:

R1. √n an =a
m
R2. √ a =( √ a )
n m n m n
=a

R3. √n a ∙ √n b=√n ab

√ a √a
n
n
R4. =
b √n b

R5. √ √ a= √ a
m n mn

Laws of Logarithms:

L1. log b MN =log b M + log b N


M
L2. log b =log b M −log b N
N

L3. log b N p =p log b N


L4. log b b=1
logb N
L5. b =N

98
College of Engineering Education
2nd Floor, BE Building
Matina Campus, Davao City
Telefax: (082)296-1084
Phone No.: (082)300-5456/300-0647 Local 131

Aside from the laws stated above, we need to introduce special logarithms that
occur on a very usual basis. Here are the definitions and notations of the logarithms:

common logarithm: log x=log 10 x


natural logarithm: ln x=log e x where e=2.718281828

2. Differentiation of Logarithmic Functions. The formulas given below provide


rules for finding the derivatives of logarithmic functions. In these formulas, u is a
function of x .

DIFFERENTIATION OF LOGARITHMIC FUNCTIONS

d 1 du
dx
( log b u )= ( log b e )
u dx

d 1 du
( ln u ) =
dx u dx

To use these formulas, study the following examples for differentiating


logarithmic functions.

dy
Example 6a.1: Find if y=log 5 (4 x+3)
dx
Solution: Let u=4 x +3

dy
=
1
dx 4 x +3
(log 5 e)
d
[
dx
(4 x +3)
]
dy 1
= (log 5 e)(4)
dx 4 x +3
dy 4 log5 e
=
dx 4 x +3

dy
Example 6a.2: Find if y=ln(2 x+ 1)4
dx
Solution: Let u=¿

dy 1
=
dx ¿ ¿

99
College of Engineering Education
2nd Floor, BE Building
Matina Campus, Davao City
Telefax: (082)296-1084
Phone No.: (082)300-5456/300-0647 Local 131

dy 1
=
dx ¿ ¿
dy 1
=
dx ¿ ¿
dy 1
=
dx ¿ ¿
dy 8
=
dx 2 x +1

Alternative Solution for Example 6a.2:


4
y=ln (2 x+ 1)
y=4 ln(2 x +1) use Law L3 to simplify the equation

dy d
=4 ∙ [ ln( 2 x +1) ]
dx dx
dy
dx
=4 ( ) [ dxd (2 x +1)]
1
2 x +1

=4 (
2 x +1 )
dy 1
(2)
dx
dy 8
=
dx 2 x +1

 You can choose to do either method since it will still result to a same answer.

Example 6a.3: Find y ' if y=ln


x +4
x−4 √
Solution: We can apply Law L3 to simplify the equation

( )
1
x +4 2
y=ln
x−4

y= ln (
x−4 )
1 x+ 4
2

1
y= ¿ use Law L2
2

y '=
{ [
1 1 d
2 x+ 4 dx
(x + 4) −
]
1 d
x−4 dx [
(x−4)
]}
100
College of Engineering Education
2nd Floor, BE Building
Matina Campus, Davao City
Telefax: (082)296-1084
Phone No.: (082)300-5456/300-0647 Local 131

y '=
1 1
[
2 x +4
(1)−
1
x−4
(1)
]
y '=
1 1
(−
1
2 x+ 4 x−4 )
y=
'
[
1 ( x−4 )− ( x + 4 )
2 ( x+ 4 )( x−4 ) ]
y '=
[
1 x−4−x−4
2 (x+ 4)(x−4) ]
y '=
1 −8
(
2 x2−16 )
−4
y '= 2
x −16
Example 6a.4: Find y ' if y=
2
√3
arccot
2 x+1
√3 ( )
Solution:

[ ( )]
[ ( √ )]
' 2 −1 d 2 x+ 1
y=
√3 1+
2 x+1
2
dx 3
√3

[ [] ( √ ) ]
' −2 1 1 d
y= ( 2 x+1 )
√3 ( 2 x +1 )2 3 dx
1+ 2
(√ 3)

( () √23 )
−2 1
y'=
√3 1+
2
4 x + 4 x+ 1
3

( )
−4 1
y'=
3 3+ 4 x 2+ 4 x +1
3

y=
' −4 3
(
3 4 x + 4 x +4
2 )
' −1
y= 2
x + x +1

101
College of Engineering Education
2nd Floor, BE Building
Matina Campus, Davao City
Telefax: (082)296-1084
Phone No.: (082)300-5456/300-0647 Local 131

Example 6a.5: Find y ' if y=ln tan ( x2 )


Solution: This problem involves a trigonometric function, thus recall the derivative of the
trigonometric function.

y'=
tan
1 d
x dx [ x
(tan )
2 ]
2

y ' =cot
x
2 (
sec 2 )[ ( )]
x d x
2 dx 2
note:
1
tan u
=cot u

y ' =cot
x
2 (
sec 2
x
2 )[( 12 ) dxd ( x ) ]
y ' =cot
x
2 (
sec 2
x
2 )[( 12 ) (1)]
y'=
1
2 ( )(
cot
x
2
sec 2
x
2 ) simplify all trig functions by using sine and cosine

( )(
x
cos

)
' 1 2 1
y=
2 x x
sin cos2
2 2

( )
' 1 1 1
y= =
2 x x x x use double-angle formula:
sin cos 2 sin cos
2 2 2 2
x x
2 sin cos =sin 2
2 2
x
2 [ ( )]
' 1
y= =csc x
sin x

3. Logarithmic Differentiation. This technique is used when the function to be


derived is expressed as a product, quotient, power, or root of two (2) or more
differentiable functions of x . This process is done by the following steps:

i. Take the natural logarithm (ln) of both sides of the equations which defines
the function.
102
College of Engineering Education
2nd Floor, BE Building
Matina Campus, Davao City
Telefax: (082)296-1084
Phone No.: (082)300-5456/300-0647 Local 131

ii. Make use of the properties and laws of logarithms to simplify the right-side
member of the equation.
dy
iii. Differentiate both sides of the equation with respect to x and solve for .
dx

dy
Example 6a.6: Find if y=(2 x+1) √ 3 x +5, using logarithmic differentiation.
dx

Solution: The equation given is a product of two differentiable functions of x, thus we will use
logarithmic differentiation to find dy /dx .

ln y =ln(2 x +1) √ 3 x +5 take ln of both sides

ln y =ln ( 2 x +1 ) + ln √ 3 x+ 5 use Law L1

1
ln y =ln ( 2 x +1 ) + ln (3 x +5) use Law L3
2

d
dx
( ln y )=
d
dx [ 1
ln ( 2 x +1 ) + ln(3 x +5)
2 ]
( )(
1 dy
y dx
=
1
2 x+ 1 )[ dxd ( 2 x +1) ]+( 12 )( 3 x1+5 ) [ dxd ( 3 x +1) ]
( )(
1 dy
y dx
=
1
2 x+ 1
(2)+
1
2) ( )( 3 x+5
1
)(3)
( )
1 dy
=
2
+
y dx 2 x +1 2
1
( )( 3 x+5
3
)
dy
dx
=y
[ 2
+
3
2 x +1 2 ( 3 x +5 ) ] transpose y to the right side of the equal sign

dy
dx
=[(2 x +1) √ 3 x +5 ] [
2 ( 2 )( 3 x +5 ) +(3)(2 x +1)
2(2 x +1)(3 x+5) ]
dy [ (2 x +1 ) √ 3 x +5 ] ( 12 x +20+6 x +3 )
= cancel similar terms from numerator and
dx 2 (2 x +1)(3 x+5)
denominator

dy 18 x+ 23
=
dx 2 √ 3 x+5

103
College of Engineering Education
2nd Floor, BE Building
Matina Campus, Davao City
Telefax: (082)296-1084
Phone No.: (082)300-5456/300-0647 Local 131

Example 6a.7: Find y ' if y=x x using logarithmic differentiation.

Solution: The equation given is a power of two differentiable functions of x so we can use
logarithmic differentiation here.

x
ln y =ln x Take natural logarithms (ln ) of both sides

ln y =x ln x use L3

d d
( ln y )= ( x ln x ) differentiate both sides
dx dx

1 d
y dx
( y )=x
1
x ()
(1 )+ ln x(1) use product rule for
d
dx
( x ln x )

dy
= y (1+ ln x ) simplify and transpose y to the right side of “=”
dx
dy x
=x (1+ ln x ) substitute the value of y which is x x
dx

By using the logarithmic differentiation, you can prove the general power formula
d n
( u ) =nun−1 du . Do it on your own and you will see how amazing it is that differentiation
dx dx
formulas are connected.

4. Differentiation of Exponential Functions. The given formulas below are used


to derive another type of transcendental function, the exponential function.

DIFFERENTIATION OF EXPONENTIAL FUNCTIONS

d u
( a )=a u ( ln a ) du
dx dx

d u
( e )=eu du
dx dx

Take note that the variable u in the formulas of the above table is a differentiable
function of x. See the examples on the next page to deeply understand the point.

104
College of Engineering Education
2nd Floor, BE Building
Matina Campus, Davao City
Telefax: (082)296-1084
Phone No.: (082)300-5456/300-0647 Local 131

Example 6a.8: Find y ' if y=4 2 x .

Solution: The base of the given function is a constant while its power has the variable x . This
means we need to use the differentiation of exponential function.

2x
y=4 where u=2 x

y '=4 2 x (ln 4 ) ( dxd 2 x )


2x
y '=4 (ln 4 ) ( 2 )
2x
y '=4 (2 ln 4)
y ' =42 x ( ln 42 ) use L3

y ' =42 x ( ln 16 )
3
Example 6a.9: Find y ' if y=e−x .

Solution:

y ' =e− x
3

[ d
dx
(−x 3 ) ]
[
y ' =e− x (−3) ( x 3−1 )
3

( dxd x )]
[ (−3)( x 3−1 ) ( 1 ) ]
3
' −x
y =e

y ' =e− x (−3 x 2 )


3

y ' =−3 x 2 e−x

Self-Help: Refer to the sources below to help you further understand the
lesson.

Feldman, J., & Rechnitzer, A. (2015). Differential Calculus Notes for Mathematics 100.
Lax, P. D., & Terell, M. S. (2014). Calculus with Applications. New York City: Springer.

105
College of Engineering Education
2nd Floor, BE Building
Matina Campus, Davao City
Telefax: (082)296-1084
Phone No.: (082)300-5456/300-0647 Local 131

Terano, H. J. (2015). Calculus 1: A simplified Text in Differential Calculus.

Let’s Check

Activity 1. Find the derivative of the following functions and simply it whenever possible.

1. y=3 4 x 6. y=ln ¿ ¿

2. y=log √ 2 x +5
x
7. y=x e
x
1+2 8. y=ln( x +3)4
3. y= x
1−2
9. y=e−4 x
4. y=log ( sin2 4 x )
10. y=x 4 ¿
5. y=4 x ln 4 x

Let’s Analyze

Activity 1. Differentiate the following functions using logarithmic differentiation.


2 1
1. y= ( 2 x +1 ) 3 ( 3 x−4 ) 2

( x−2 )3 √ x+1
2. y= 4
( x−3 )

3. y=

3 ( 4 x −1 )( x +2 )
( x−5 )
2

Activity 2. Find the derivative of the following functions and simplify whenever possible.
6. y=ln(x + √ x2 +1)
x
1. y=x 2

2. y=e x e ln x 7. y=ln 2 (x +3)

106
College of Engineering Education
2nd Floor, BE Building
Matina Campus, Davao City
Telefax: (082)296-1084
Phone No.: (082)300-5456/300-0647 Local 131

( )
3
3. y=ln (e x x 2) 1−sin x
8. y=ln
1+sin x
2x
e −1
4. y=ln 2x
e +1 9. y=x arctan x−ln √ 1+ x 2

x+y x 10. y=ln ¿ ¿


5. e =ln
y

In a Nutshell
You need to take note of these points when dealing with the differentiation of
logarithmic and exponential functions.

 The graph of a logarithmic function is always on the positive side of the y-axis.
 The graph of an exponential function never crosses the x-axis and always
intersects the y-axis at y=1 .
 There are properties and laws of logarithms and exponents that will help simplify
the functions process of differentiation.
 Logarithmic Differentiation is a method used when an equation to be derived is
expressed as a product, quotient, root, or power of two (2) or more differentiable
functions of x.

Q&A List

If you have any questions regarding continuity, kindly write down on the table
provided.

QUESTIONS ANSWERS
1.

2.

3.

4.

107
College of Engineering Education
2nd Floor, BE Building
Matina Campus, Davao City
Telefax: (082)296-1084
Phone No.: (082)300-5456/300-0647 Local 131

5.
Big Picture in Focus:
ULO-6b. Familiarize the properties of the hyperbolic functions and find its derivative

Metalanguage
In this section, you will find terms that can be used when dealing with hyperbolic
functions. You may go back here anytime if you run across a certain term you find
ambiguous.
1. Hyperbolic Functions. These functions are analogs of the standard trigonometric
functions which are defined for the hyperbola rather than on the circle.
1.1 These functions are sometimes called the “hyperbolic trigonometric functions”
since it has many connections between them and the standard trigonometric
functions.
1.2 These functions relate to the hyperbola same as the trigonometric functions
relate to the circle.
Essential Knowledge
In ULO6b, you are expected to familiarize the properties of hyperbolic functions
for its easy derivation. This function is the last transcendental function which will be
discussed.
1. The Hyperbolic Functions. These functions are a combination of certain
exponential functions, e x and e− x. Hyperbolic functions are defined as follows:

DEFINITIONS OF HYPERBOLIC FUNCTIONS


x −x
e −e cosh x
sinh x= coth x=
2 sinh x
x −x
e +e 1
cosh x= sech x=
2 cosh x

sinh x 108 1
tanh x= csch x =
cosh x sinh x
College of Engineering Education
2nd Floor, BE Building
Matina Campus, Davao City
Telefax: (082)296-1084
Phone No.: (082)300-5456/300-0647 Local 131

The notation sinh x which is pronounced as “shine” can be read “hyperbolic sine
of x”. The others can also be read in the same manner.

Like the trigonometric functions, hyperbolic functions also have its identities
which are illustrated on a table on the next page. These identities can be directly
deduced from the definitions of hyperbolic functions.

HYPERBOLIC IDENTITIES

H1. cosh 2 x−si nh2 x=1

H2. tanh2 x + sec h2 x=1

H3. coth2 x−csch 2 x=1

H4. sinh 2 x=2 sinh x cosh x

H5. cosh 2 x=cosh 2 x +sinh2 x


2
¿ 1+2 sinh x
2
¿ 2 cosh x−1

The hyperbolic identities would come in handy later as we start differentiating the
hyperbolic functions.

2. Differentiation of Hyperbolic Functions. The rules for differentiating the hyperbolic


functions are given by the following formulas where u is a function of x .

DIFFERENTIATION OF HYPERBOLIC FUNCTIONS

d du d du
( sinh u )=cosh u ( coth u )=−c sch2 u
dx dx dx dx

d du d du
( cosh u )=sinh u ( sech u )=−sech u tanhu
dx dx dx dx

d du d du
( tanhu )=sech 2 u ( csch u )=−csch u cothu
dx dx dx dx

109
College of Engineering Education
2nd Floor, BE Building
Matina Campus, Davao City
Telefax: (082)296-1084
Phone No.: (082)300-5456/300-0647 Local 131

Study the examples on the succeeding page for you to know how to use the
formulas in differentiating hyperbolic functions.

dy
Example 6b.1: Find if y=sinh ( 4 x +3 ) .
dx
Solution:
dy
dx
=cosh ( 4 x +3 )
d
dx [
( 4 x+3 ) ]
dy
=cosh ( 4 x +3 ) ( 4)
dx

dy
=4 cosh ( 4 x+ 3 )
dx

Example 6b.2: Find y ’ if y=3 cosh 2 4 x

Solution:
y ' =( 3 ) ( 2 ) ( cosh 2−1 4 x ) ¿

y ' =6 cosh 4 x sinh 4 x ( dxd 4 x )


'
y =24 cosh 4 x sinh 4 x

This is not yet the final answer since we can still simplify it using the hyperbolic identity
sinh 2 x=2 sinh x cosh x.
'
y =12 ¿ where x=4 x
'
y =12 sinh 2(4 x)
'
y =12 sinh 8 x

Example 6b.3: Find y ’ if y=sinh(ln x )


Solution:
y ' =( cosh ln x ) ¿

y ' =( cosh ln x ) ( 1x )
110
College of Engineering Education
2nd Floor, BE Building
Matina Campus, Davao City
Telefax: (082)296-1084
Phone No.: (082)300-5456/300-0647 Local 131

We can simplify the equation further by applying one of the definitions of hyperbolic function--
x −x
e +e
cosh x=
2

( )( 1x )
ln x −ln x
' e +e where x=ln x
y=
2
ln x −ln x
' e +e
y= .
2x
−ln x ln x−1 −1 1
Use the logarithmic identity: e ln x =x and e =e =x = to simplify further..
x
1
x+ 2
x x + 1.
y'= =
2x 2x
2

Self-Help: Refer to the sources below to help you further understand the
lesson.

Feldman, J., & Rechnitzer, A. (2015). Differential Calculus Notes for Mathematics 100.
Terano, H. J. (2015). Calculus 1: A simplified Text in Differential Calculus.

Let’s Check

Activity 1. Find the derivative of the following hyperbolic functions and simplify whenever
possible.

1. y=sinh2 5 x

2. y=arctan ¿

3. y=ln ¿ ¿

sinh x
4. y=
1+cosh x

5. y=tanh x 2

Let’s Analyze

Activity 1. Differentiate the following functions and simplify if possible.

111
College of Engineering Education
2nd Floor, BE Building
Matina Campus, Davao City
Telefax: (082)296-1084
Phone No.: (082)300-5456/300-0647 Local 131

1
1. y=coth
x
2. y=csch2 3 x
3. y=s ech2 ln x
4. y=sinh x cosh x−x

x
5. y= ¿
2

6. y=arctan ¿

7. y=e x ln¿ ¿

2 1
8. y=cosh 6 x + cosh 12 x
2

In a Nutshell
You need to take note of these points when dealing with the differentiation of
hyperbolic functions.

 Hyperbolic function is a combination of the exponential functions e x and e− x .


 The definitions and identities of hyperbolic functions are necessary to simplify the
equation.
 Differentiation of hyperbolic functions have rules, which are expressed in
formulas, that should be followed to avoid using the long method.

Q&A List

If you have any questions regarding continuity, kindly write down on the table
provided.

QUESTIONS ANSWERS
1.

2.

3.

112
College of Engineering Education
2nd Floor, BE Building
Matina Campus, Davao City
Telefax: (082)296-1084
Phone No.: (082)300-5456/300-0647 Local 131

4.

5.

Big Picture in Focus:


ULO-7a. Solve Parametric Equations: curve tracing

Big Picture
Week 13-14: Unit Learning Outcomes 7 (ULO 7): At the end of the unit, you are
expected to:
a. Solve Parametric Equations: curve tracing,
b. Solve for Partial derivatives,

Metalanguage
Instead of a function y(x) being defined explicitly in terms of the independent
variable x, it is sometimes useful to define both x and y in terms of a third variable, t
say, known as a parameter. In this unit we explain how such functions can be
differentiated using a process known as parametric differentiation.

113
College of Engineering Education
2nd Floor, BE Building
Matina Campus, Davao City
Telefax: (082)296-1084
Phone No.: (082)300-5456/300-0647 Local 131

In order to master the techniques explained here, it is vital that you undertake
plenty of practice exercises so that they become second nature.
After reading this text, and/or viewing the video tutorial on this topic, you should
be able to:
• differentiate a function defined parametrically
• find the second derivative of such a function

Essential Knowledge
To perform the aforesaid big picture (unit learning outcomes), you need to fully
understand the following essential knowledge that will be laid down in the succeeding
pages. Please note that you are not limited to exclusively refer to these resources.
Thus, you are expected to utilize other books, research articles and other resources that
are available in the university’s library (refer to the Library Contact on page 3).

It is often necessary to find the rate of change of a function defined parametrically; that
dy
is, we want to calculate . The following example will show how this is achieved.
dx

dy
Example: Suppose we wish to find when x = cost and y = sint.
dx
We differentiate both x and y with respect to the parameter, t:
dx dy
=−sint =cost
dt dt

From the chain rule we know that,


dy dy dx
= .
dt dx dt

So that, by rearrangement
dy
dy dt cost
= = =−cot t
dx dx −sint
dt

114
College of Engineering Education
2nd Floor, BE Building
Matina Campus, Davao City
Telefax: (082)296-1084
Phone No.: (082)300-5456/300-0647 Local 131

dy
EXAMPLE 1: Suppose we wish to find when x=t 3−t∧ y=4−t 2.
dx

dy
EXAMPLE 2: Suppose we wish to find when x=t 3∧ y=t 2−t .
dx
115
College of Engineering Education
2nd Floor, BE Building
Matina Campus, Davao City
Telefax: (082)296-1084
Phone No.: (082)300-5456/300-0647 Local 131

116
College of Engineering Education
2nd Floor, BE Building
Matina Campus, Davao City
Telefax: (082)296-1084
Phone No.: (082)300-5456/300-0647 Local 131

SECOND DERIVATIVES

117
College of Engineering Education
2nd Floor, BE Building
Matina Campus, Davao City
Telefax: (082)296-1084
Phone No.: (082)300-5456/300-0647 Local 131

118
College of Engineering Education
2nd Floor, BE Building
Matina Campus, Davao City
Telefax: (082)296-1084
Phone No.: (082)300-5456/300-0647 Local 131

Self-Help: Refer to the sources below to help you further understand the
lesson.

Love, C., & Rainville E., (1962). Differential Calculus Notes for Mathematics 100.
Feliciano F., and Uy, F., (1983). Differential Calculus and Integral Calculus.
e-source
http://www.mathcentre.ac.uk/

Let’s Check

1.

2.

119
College of Engineering Education
2nd Floor, BE Building
Matina Campus, Davao City
Telefax: (082)296-1084
Phone No.: (082)300-5456/300-0647 Local 131

Let’s Analyze

1.

In a Nutshell

For second derivative,

Q&A List

If you have any questions regarding this topic, kindly write down on the table
provided.

QUESTIONS ANSWERS
1.

2.

3.

4.

5.

120
College of Engineering Education
2nd Floor, BE Building
Matina Campus, Davao City
Telefax: (082)296-1084
Phone No.: (082)300-5456/300-0647 Local 131

Big Picture in Focus:


ULO-7b. Solve for Partial derivatives

Metalanguage
Before proceeding to “Essential Knowledge”, you should already master the
principle of differentiating a function of one variable. I believe you have already learned
that in the previous units. To be able to catch up to the lesson, please refer to the
following definition of terms.

A partial derivative of a function of several variables is its derivative with respect to


one of those variables, with the others held constant (as opposed to the total derivative,
in which all variables are allowed to vary). The symbol used to denote partial derivatives
is ∂ .
An independent variable is a variable that stands alone and isn't changed by the other
variables you are trying to measure. For equations such as y = 3x – 2, the independent
variable is x. The variable y is not independent since it depends on the number chosen
for x.
A dependent variable is a variable that depends on one or more other variables. For
equations such as y = 3x – 2, the dependent variable is y.

Essential Knowledge
When a function of more than one independent input variable changes because
of changes in one or more of the input variables, it is important to calculate the change
in the function itself. This can be investigated by holding all but one of the variables
constant and finding the rate of change of the function with respect to the one remaining
variable. This process is called partial differentiation. In this section, we show how to
carry out the process.

FIRST PARTIAL DERIVATIVES

The x partial derivative


For a function of a single variable, y=f(x), changing the independent variable x leads to
a corresponding change in the dependent variable y. The rate of change of y with
df
respect to x is given by the derivative, written . A similar situation occurs with
dx
functions of more than one variable. For clarity we shall concentrate on functions of just
two variables.

In the relation z=f(x, y) the independent variables are x and y and the dependent
variable z. Now, both variables x and y may change simultaneously inducing a change
in z. However, rather than consider this general situation, to begin with we shall hold

121
College of Engineering Education
2nd Floor, BE Building
Matina Campus, Davao City
Telefax: (082)296-1084
Phone No.: (082)300-5456/300-0647 Local 131

one of the independent variables fixed. This is equivalent to moving along a curve
obtained by intersecting the surface by one of the coordinate planes.
Consider f(x, y) =x3 + 2x2y+ y2+ 2x+ 1.

Suppose we keep y constant and vary x; then what is the rate of change of the function
f?

Suppose we hold y at the value 3 then


f(x,3) =x3+ 6x2+ 9 + 2x + 1 = x3 + 6x2+ 2x+ 10

In effect, we now have a function of x only. If we differentiate it with respect to x we


obtain the expression:
3x2 + 12x+ 2.

We say that f has been partially differentiated with respect tox. We denote the partial
∂f
derivative of f with respect to x by (to be read as ‘partial dee f by dee x’). In this
∂x
example when y= 3:
∂f
= 3x2+ 12x+ 2
∂x

In the same way if y is held at the value 4 then f(x,4) =x3 + 8x2 + 16 + 2x + 1 = x3 + 8x2 +
2x + 17and so, for this value of y
∂f
= 3x2+ 16x+ 2
∂x

Now if we return to the original formulation


f(x, y) = x3 + 2x2y + y2 + 2x + 1
and treat y as a constant then the process of partial differentiation with respect to x
gives
∂f
= 3x2+ 4xy + 0 + 2
∂x
∂f
= 3x2+ 4xy + 2
∂x

122
College of Engineering Education
2nd Floor, BE Building
Matina Campus, Davao City
Telefax: (082)296-1084
Phone No.: (082)300-5456/300-0647 Local 131

EXAMPLE:

a.)

Here, we treat y as constant. Therefore,


∂f 2
=3 x +1+0+0
∂x
∂f 2
=3 x +1
∂x

b.)

Variable y is constant here. Therefore,

∂f 3
=2 x (0)+ y (2 x)+ x (0)+ y (1)
∂x
∂f 3
=2 xy + y
∂x

EXAMPLE:

123
College of Engineering Education
2nd Floor, BE Building
Matina Campus, Davao City
Telefax: (082)296-1084
Phone No.: (082)300-5456/300-0647 Local 131

∂f
We can calculate the partial derivative of f with respect to x and the value of at a
∂x
specific point e.g. x= 1, y=−2.

EXAMPLE:

Functions of several variables

124
College of Engineering Education
2nd Floor, BE Building
Matina Campus, Davao City
Telefax: (082)296-1084
Phone No.: (082)300-5456/300-0647 Local 131

Second partial derivatives

EXAMPLE:

We can use the alternative notation when evaluating derivatives.

EXAMPLE:

125
College of Engineering Education
2nd Floor, BE Building
Matina Campus, Davao City
Telefax: (082)296-1084
Phone No.: (082)300-5456/300-0647 Local 131

Mixed second derivatives

EXAMPLE 1:

EXAMPLE 2:

126
College of Engineering Education
2nd Floor, BE Building
Matina Campus, Davao City
Telefax: (082)296-1084
Phone No.: (082)300-5456/300-0647 Local 131

Self-Help: Refer to the sources below to help you further understand the
lesson.

Helm (2008). Workbook 18: Functions of Several Variables


Feliciano F., and Uy, F., (1983). Differential Calculus and Integral Calculus.
Miller, Jeff (2009-06-14). "Earliest Uses of Symbols of Calculus". Earliest Uses of Various
Mathematical Symbols. Retrieved 2009-02-20

Let’s Check

Let’s Analyze

127
College of Engineering Education
2nd Floor, BE Building
Matina Campus, Davao City
Telefax: (082)296-1084
Phone No.: (082)300-5456/300-0647 Local 131

In a Nutshell

Q&A List

If you have any questions regarding this topic, kindly write down on the table
provided.

QUESTIONS ANSWERS
1.

2.

3.

128
College of Engineering Education
2nd Floor, BE Building
Matina Campus, Davao City
Telefax: (082)296-1084
Phone No.: (082)300-5456/300-0647 Local 131

4.

Big Picture
Week 15-16: Unit Learning Outcomes 8 (ULO 8): At the end of the unit, you are
expected to:

Big Picture in Focus:


ULO-8a. Learn Approximate limits that produce indeterminate form and use
L`Hospitals Rule to evaluate limits

a. Learn Approximate limits that produce indeterminate form and use L`Hospitals
Rule to evaluate limits
b. Learn and solve for solutions of equations: Newton’s Method.

Metalanguage
To be able to catch up to the lesson, please familiarize yourself to the following
definition of terms:

A mathematical expression can also be said to be indeterminate if it is not definitively


or precisely determined. Certain forms of limits are said to be indeterminate when
merely knowing the limiting behavior of individual parts of the expression is not sufficient
to actually determine the overall limit.

An indeterminate form is an expression involving two functions whose limit cannot be


determined solely from the limits of the individual functions. These forms are common in
calculus; indeed, the limit definition of the derivative is the limit of an indeterminate form.

Types if indeterminate forms:

1. 0/0 3. 0x∞ 5. 00 7. ∞ 0
2. ∞ /∞ 4. ∞−∞ 6. 1∞

Essential Knowledge
In the previous unit, you studied limits such as

129
College of Engineering Education
2nd Floor, BE Building
Matina Campus, Davao City
Telefax: (082)296-1084
Phone No.: (082)300-5456/300-0647 Local 131

In those sections, you discovered that direct substitution can produce an indeterminate
form such as 0/0 or ∞ /∞ . For instance, if you substitute x=1 into the first limit, you
obtain

which tells you nothing about the limit. To find the limit, you can factor and divide out
like factors, as shown.

For the second limit, direct substitution produces the indeterminate form ∞ /∞ , which
again tells you nothing about the limit. To evaluate this limit, you can divide the
numerator and denominator by x. Then you can use the fact that the limit of 1/x, as x
approaches ∞ is 0.

Algebraic techniques such as these tend to work well as long as the function itself is
algebraic. To find the limits of other types of functions, such as exponential functions or
trigonometric functions, you generally need to use a different approach.

130
College of Engineering Education
2nd Floor, BE Building
Matina Campus, Davao City
Telefax: (082)296-1084
Phone No.: (082)300-5456/300-0647 Local 131

L’Hôpital’s Rule

L’Hôpital’s Rule, which is named after the French mathematician Guillaume Francois
Antoine de L’Hôpital (1661–1704), describes an analytic approach for evaluating limits.

EXAMPLE 1:

Because direct substitution produces the indeterminate form 0/0, you can apply
L’Hôpital’s Rule to obtain the same result.

EXAMPLE 2:

131
College of Engineering Education
2nd Floor, BE Building
Matina Campus, Davao City
Telefax: (082)296-1084
Phone No.: (082)300-5456/300-0647 Local 131

Again, because direct substitution produces the indeterminate form 0/0, you can apply
L’Hôpital’s Rule to obtain the same result.

EXAMPLE 3:

Because direct substitution produces the indeterminate form ∞ /∞ .

Sometimes it is necessary to apply L’Hôpital’s Rule more than once to remove an


indeterminate form. This is shown in Example 4.

132
College of Engineering Education
2nd Floor, BE Building
Matina Campus, Davao City
Telefax: (082)296-1084
Phone No.: (082)300-5456/300-0647 Local 131

EXAMPLE 4:

Self-Help: Refer to the sources below to help you further understand the
lesson.

Feliciano F., and Uy, F., (1983). Differential Calculus and Integral Calculus.
133
College of Engineering Education
2nd Floor, BE Building
Matina Campus, Davao City
Telefax: (082)296-1084
Phone No.: (082)300-5456/300-0647 Local 131

https://www.math.ucdavis.edu/~marx/Sec.%208.6.pdf

Let’s Check

Decide whether the limit produces an indeterminate form.

Let’s Analyze
Use L’Hôpital’s Rule to find the limit. You mayneed to use L’Hôpital’s Rule repeatedly.

1.

2.

3.

4.

5.

6.

7.

9.

10.

11.
134
College of Engineering Education
2nd Floor, BE Building
Matina Campus, Davao City
Telefax: (082)296-1084
Phone No.: (082)300-5456/300-0647 Local 131

12.

In a Nutshell

Q&A List

If you have any questions regarding this topic, kindly write down on the table
provided.

QUESTIONS ANSWERS
1.

2.

3.

4.

5.

135
College of Engineering Education
2nd Floor, BE Building
Matina Campus, Davao City
Telefax: (082)296-1084
Phone No.: (082)300-5456/300-0647 Local 131

Big Picture in Focus:


ULO-8b. Learn and solve for solutions of equations: Newton’s Method

Metalanguage
To be able to catch up to the lesson, please familiarize yourself to the following
definition of terms:

Newton's Method (also called the Newton-Raphson method) is a recursive algorithm


for approximating the root of a differentiable function.

Iteration is the repetition of a mathematical or computational procedure applied to the


result of a previous application, typically as a means of obtaining successively closer
approximations to the solution of a problem.

Essential Knowledge
The next application that we’ll take a look at in this unit is an important application that is
used in many areas. If you’ve been following along to this point it’s quite possible that
you’ve gotten the impression that many of the applications that we’ve looked at are just
made up by us to make you work. This is unfortunate because all of the applications
that we’ve looked at to this point are real applications that really are used in real
situations. The problem is often that in order to work more meaningful examples of the
applications we would need more knowledge than we generally have about the science
and/or physics behind the problem. Without that knowledge we’re stuck doing some
fairly simplistic examples that often don’t seem very realistic at all and that makes it
hard to understand that the application we’re looking at is a real application.

That is going to change in this section. This is an application that we can all understand,
and we can all understand needs to be done on occasion even if we don’t understand
the physics/science behind an actual application.

In this section, we are going to look at a method for approximating solutions to


equations. We all know that equations need to be solved on occasion and in fact we’ve
solved quite a few equations ourselves to this point. In all the examples we’ve looked at
to this point we were able to actually find the solutions, but it’s not always possible to do
that exactly and/or do the work by hand. That is where this application comes into play.
So, let’s see what this application is all about.

136
College of Engineering Education
2nd Floor, BE Building
Matina Campus, Davao City
Telefax: (082)296-1084
Phone No.: (082)300-5456/300-0647 Local 131

Let’s suppose that we want to approximate the solution to f (x)=0 and let’s also
suppose that we have somehow found an initial approximation to this solution say, x 0.
This initial approximation is probably not all that good, in fact it may be nothing more
than a quick guess we made, and so we’d like to find a better approximation. This is
easy enough to do. First, we will get the tangent line to f (x) at x 0.

Now, take a look at the graph below.

The blue line is the tangent line at x 0. We can see that this line will cross the x-axis
much closer to the actual solution to the equation than x 0 does. Let’s call this point
where the tangent at x 0 crosses the x-axis x 1 and we’ll use this point as our new
approximation to the solution.

So, how do we find this point? Well we know it’s coordinates, ( x 1 , 0 ¿, and we know that
it’s on the tangent line so plug this point into the tangent line and solve for x 1 as follows,

0=f (x 0)+ f ' (x 0)(x 1−x 0 )


−f ( x 0)
x 1−x 0=
f ' (x 0)
f (x 0)
x 1=x 0−
f ' (x 0)

So, we can find the new approximation provided the derivative isn’t zero at the original
approximation.

Now we repeat the whole process to find an even better approximation. We form up the
tangent line to f(x) at x 1 and use its root, which we’ll call x 2, as a new approximation to
the actual solution. If we do this, we will arrive at the following formula.

137
College of Engineering Education
2nd Floor, BE Building
Matina Campus, Davao City
Telefax: (082)296-1084
Phone No.: (082)300-5456/300-0647 Local 131

f (x 1 )
x 2=x 1−
f '(x 1 )
This point is also shown on the graph above and we can see from this graph that if we
continue following this process will get a sequence of numbers that are getting very
close the actual solution. This process is called Newton’s Method.

Newton’s Method

This should lead to the question of when do we stop? How many times do we go
through this process? One of the more common stopping points in the process is to
continue until two successive approximations agree to a given number of decimal
places.

Before working any examples, we should address two issues. First, we really do need to
be solving f(x)=0 in order for Newton’s Method to be applied. This isn’t really all that
much of an issue, but we do need to make sure that the equation is in this form prior to
using the method.

Secondly, we do need to somehow get our hands on an initial approximation to the


solution (i.e. we need x 0 somehow). One of the more common ways of getting our
hands on x 0 is to sketch the graph of the function and use that to get an estimate of the
solution which we then use as x 0. Another common method is if we know that there is a
solution to a function in an interval then we can use the midpoint of the interval as x 0.

Let’s work an example of Newton’s Method.

EXAMPLE 1: Use Newton’s Method to determine x2 for f(x)=x3−7x2+8x−3 if x0=5.


f (x n )
x n+1=x n −
f ' (x n )
So all we need to do is run through this twice.

Here is the derivative of the function since we’ll need that.


2
f ’ ( x )=3 x −14 x+ 8

The first iteration through the formula for x1 is,


f (x 0) f (5) −13
x 1=x 0− =5− =5− =6
f ' (x 0) f ' (5) 13

138
College of Engineering Education
2nd Floor, BE Building
Matina Campus, Davao City
Telefax: (082)296-1084
Phone No.: (082)300-5456/300-0647 Local 131

The second iteration through the formula for x2 is,


f (x 1 ) f (6) 9
x 2=x 1− =6− =6− =5.71875
f '(x 1 ) f ' (6) 32

So, the answer for this problem is 5.71875.

EXAMPLE 2: Use Newton’s Method to determine x2 for f(x) = xcos(x) − x2 if x0 = 1.


f (x n )
x n+1=x n −
f ' (x n )
Here is the derivative of the function since we’ll need that.
f ' (x)=cos( x)−xsin(x )−2 x
The first iteration through the formula for x1 is,
f (x 0) f (1) −0.4596976941
x 1=x 0− =1− =1− =0.8002329432
f ' (x 0) f ' (1) −2.301168679

Important!!! Don’t forget that for us angles are always in radians so make sure
your calculator is set to compute in radians.

The second iteration through the formula for x2 is,


f (x 1 ) f (0.8002329432)
x 2=x 1− =0.8002329432− =0.7440943985
f '(x 1 ) f '( 0.8002329432)

So, the answer for this problem is 0.7440943985 .

EXAMPLE 3: Use Newton’s Method to determine an approximation to the solution to


cosx =x that lies in the interval [0,2]. Find the approximation to six decimal places.

First note that we weren’t given an initial guess. We were however, given an interval in
which to look. We will use this to get our initial guess. As noted above the general rule
of thumb in these cases is to take the initial approximation to be the midpoint of the
interval. So, we’ll use x 0=1 as our initial guess.

Next, recall that we must have the function in the form f (x)=0. Therefore, we first
rewrite the equation as,
cosx −x=0

139
College of Engineering Education
2nd Floor, BE Building
Matina Campus, Davao City
Telefax: (082)296-1084
Phone No.: (082)300-5456/300-0647 Local 131

We can now write down the general formula for Newton’s Method. Doing this will often
simplify up the work a little so it’s generally not a bad idea to do this.

Let’s now get the first approximation.

At this point we should point out that the phrase “six decimal places” does not mean just
get x 1 to six decimal places and then stop. Instead it means that we continue until two
successive approximations agree to six decimal places.

Given that stopping condition we clearly need to go at least one step farther.

Alright, we’re making progress. We’ve got the approximation to 1 decimal place. Let’s
do another one, leaving the details of the computation to you.

We’ve got it to three decimal places. We’ll need another one.

And now we’ve got two approximations that agree to 9 decimal places and so we can
stop. We will assume that the solution is approximately x 4 =0.7390851332.

Self-Help: Refer to the sources below to help you further understand the
lesson.

Dawkins, P., Paul's Online Notes. Copyright 2003-2020


https://tutorial.math.lamar.edu/Solutions/CalcI/NewtonsMethod/Prob5.aspx

Let’s Check

For problems 1 – 3 use Newton’s Method to determine x2 for the given function and
given value of x0.

140
College of Engineering Education
2nd Floor, BE Building
Matina Campus, Davao City
Telefax: (082)296-1084
Phone No.: (082)300-5456/300-0647 Local 131

For problems 4 – 8 use Newton’s Method to find the root of the given equation, accurate
to six decimal places, that lies in the given interval.

Let’s Analyze

For problems 9 – 12 use Newton’s Method to find all the roots of the given equation
accurate to six decimal places.

13. Suppose that we want to find the root to x3−7x2+8x−3=0. Is it possible to use x0=4
as the initial point? What can you conclude about using Newton’s Method to
approximate roots from this example?

In a Nutshell
The approximation formula is given by,
f (x n )
x n+1=x n −
f ' (x n )

Where n is the no. of iterations. We need to be a little careful with Newton’s method. It
will usually quickly find an approximation to an equation. However, there are times when
it will take a lot of work or when it won’t work at all.

Q&A List

141
College of Engineering Education
2nd Floor, BE Building
Matina Campus, Davao City
Telefax: (082)296-1084
Phone No.: (082)300-5456/300-0647 Local 131

If you have any questions regarding this topic, kindly write down on the table
provided.

QUESTIONS ANSWERS
1.

2.

3.

4.

5.

142

You might also like